SlideShare a Scribd company logo
1 of 409
Numerical   Reasoning
Problem on Numbers
Problems on Numbers
Arithmetic Progression:
 The nth term of A.P. is given by Tn = a + (n – 1)d
  Sum of n terms of A.P S = n/2 *[2a+(n-1)d)]

Geometrical Progression:
    Tn = arn – 1.
   Sn = a(rn – 1) / (r-1)
Basic Formulae
1. ( a+b)2 = a2 + b2 + 2ab
2. (a-b)2 = a2 +b2 -2ab
3. ( a+b)2 - (a – b)2 = 4ab
4. (a+b)2 + (a – b)2 = 2 (a2 +b2)
5. (a2 – b2) = (a+b) (a-b)
6. (a+b+c)2 =a2 +b2 +c2 + 2(ab +bc+ca)
7. (a3 +b3) = ( a+b) (a2 –ab +b2)
8. (a3 –b3) = (a-b) (a2 +ab + b2)
Problem - 1
A 2 digit number is 3 times the sum of its digits
if 45 is added to the number. Its digits are
interchanged. The sum of digits of the number
is?
Solution
The number is 3 times the sum of its digits
45 is added = 4 +5 = 9
So, common numbers in 3 and 9th table.
9, 18, 27, 36, 45….
27 + 45 = 72
2 + 7 = 9 or 4 + 5 = 9
Problem - 2
A number when divided by 119 leaves a
remainder of 19. If it is divided by 17. It will
leave a remainder of?
Solution
= 19/17 = 2 remainder
Problem - 3
A boy was asked to find the value of 3/8 of sum of
money instead of multiplying the sum by 3/8 he
divided it by 8/3 and then his answer by Rs.55.
Find the correct answer?
Solution
8/3 – 3/8 = 55/24
          = 55/55/24
          = 24
Problem - 4
A man spends 2/5rd of his earning. 1/4th of the
expenditure goes to food, 1/5th on rent, 2/5th on
travel and rest on donations. If his total earning
is Rs.5000, find his expenditure on donations?
Solution
5000*2/5 = 2000
Remaining amount has given as donation
2000* (1/5 + 2/5 + ¼)
Total amount = 200*17/20 = 1700
2000 – 1700 = 300
Problem - 5
From a group of boys and girls 15 girls leave.
There are then left, 2 boys for each girl. After
this 45 boys leave, there are then left 5 girls for
each boy, find the number of girls in the
beginning?
Solution
15 girls leave = 2 boys for each girl
45 boys leave = 5 girls fro 1 boy
Let the boys be x; Girls = x/2 +15
After the boys have left,
No.of boys = x – 45 and girls = 5(x-45)
x/2 = 5(x-45)
X = 2(5x-22)
X = 10x – 450
X =50
50/2 +15 =40
Problem - 6
An organization purchased 80 chairs fro
Rs.9700. For chairs of better quality they paid
Rs.140 each and for each of the lower grade
chair they paid Rs.50 less. How many better
quality chairs did the organization buy?
Solution
Better quality chairs = x;
Lower quality = 80 –x
Price of better quality = Rs.140, Lower quality =
  140-50 = 90
140*x + 90(80-x) = 9700
140x + 7200 – 90x = 9700
50x = 9700 – 7200;
50x = 2500
X = 50
Problem - 7
A labour is engaged for 30 days, on the condition
that Rs.50 will be paid for everyday he works
and Rs.15 will be deducted from his wages for
everyday he is absent from work. At the end of
30 days he received Rs.850 in all. For how many
days did he wanted?
Solution
Total wages = 30*50 = 1500 (without Absent)
Wages received in 30 days = 850 (with Absent)
Let the labourer work for x days
Absent = 30 – x
50x – (30-x)15 = 850
50x -450 +15x = 850
65x = 1300
X = 1300/65 = 20 days
Problem - 8
The rent is charged at Rs.50 per day for first 3
days Rs.100 per day next 5 days, and 300 per
day thereafter. Registration fee is 50 at the
beginning. If a person had paid Rs.1300 for his
stay how many days did he stay?
Solution
3 days = 150 + 50 = 200
5 days = 100*5 = 500
       = 200 + 500 = 700
1300 – 700 = 600
2 days = 300*2 = 600
= 5 + 3 + 2 = 10 days
Problem - 9
In a school 20% of students are under the age of
8 years. The number of girls above the age of 8
years is 2/3 of the number of boys above the age
of 8 years and amount to 48. What is the total
number of students in the school?
Solution
Girls above 8 yrs = 48
Boys above 8 yrs = 48 / 2/3
80% of students above 8 yrs = 48 + 72 = 120
  80       120
   20       x
80x = 120*20
X = 120*20/80 = 30
Total No.of students = 120+30 = 150
Ratio and Proportion
Ratio and Proportion
Ratio : Relationship between two variables.
     =a:b
Proportion : Relationship between two ratios.
     =a:b::c:d
Proportion Calculation = a*d : b*c
Problem - 1
The ratio of number of boys to that of girls in a
school is 3:2. If 20% boys and 25% of girls are
scholarship holders, find the percentage of the
school students who are not scholarship holders?
Solution
Let the total number of students be 100
Boys = 100*3/5 = 60
Girls = 100*2/5 = 40
S. holders = 60*20/100 = 12, non S. holders = 60 -12 = 48
Girls s. holders = 40*25/100 = 10,
Non s. holders = 40 – 10 = 30
Students who do not have scholarship = 48 + 30 = 78
78/100*100 = 78%
Problem - 2
The cost of diamond varies as the square of its
weight. A diamond weighing 10 decigrams costs
Rs. 32000. Find the loss incurred when it breaks
into two pieces whose weights are in the ratio
2:3?
Solution
1st piece = 10*2/5 = 4
2nd piece = 10*3/5 = 6
Cost of the diamond varies as square of its weight
42 : 62                  102 = 100k
16k : 36k
100k – 52 k = 48k(loss)
100k = 32000; k = 320
48*320 = 15360
Problem - 3
The ratio of the first and second class fares
between two railway stations 4 : 1 and the ratio
of the number of passengers traveling by first
and second class is 1:40. If the total of Rs.1100
is collected as fare from passengers of both
classes what was the amount collected from first
class passengers?
Solution
Fare = 4 : 1
Passengers traveling = 1 : 40
Amount = No. pas * fare = 4*1 :10*1 = 4 : 40
= 1:10
Total amount = 1100.
First class passengers‟ amount = 1*1100/11
= 100
Problem - 4
A vessel contains a mixture of water and milk in
the ratio 1:2 and another vessel contains the
mixture in the ratio 3:4. Taking 1 kg each from
both mixtures a new mixture is prepared. What
will be the ratio of water and milk in the new
mixture?
Solution
1st vessel = water = 1/3 , milk = 2/3
2nd vessel = water = 3/7, milk = 4/7
Water = 1/3 + 3/7 = 16/21
Milk = 2/3 + 4/7 = 26/21
16 : 26 = 8:13
Problem - 5
Ratio of the income of A, B, C last year 3 : 4 : 5.
The ratio of their individual incomes of last year
and this year are 4:5, 2:3 and 3:4 respectively. If
the sum of their present income is Rs.78,800.
Find the present individual income of A, B and
C.
Solution
A‟s Present Income = 5/4*3x = 15x/4
B‟s Present Income = 3/2*4x = 12x/2
C‟s Present Income = 4/7*5x = 20x/7
15x/4 + 6x+20x/3 = 78,800
197x/12 = 78,800
X = 945600/197
X = 4,800
A‟s Present income = 15x/4 = 15*4800/4 = 18,000
B‟s Present income = 6*x = 6*4,800 = 28,800
C‟s Present income = 20x/3 = 20*4800/3 = 32,000
Problem - 6
Of the three numbers, the ratio of the first and
the second is 8:9 and that of the second and third
is 3:4. If the product of the first and third
numbers is 2,400, then find the second number?
Solution
a:b=8:9
b:c=3:4

b : c = 3*3 : 4*3 = 9 : 12
a : b : c = 8 : 9 : 12
Product of first and third = 8k * 12k = 2400
96k2 = 2400; k2 = 2400/96 = 25
k=5
Second number = 9 * 5 = 45
Problem - 7
Annual income of A and B are in the ratio of 4 : 3
and their annual expenses are in the ratio 3 : 2. If
each of them saves Rs.600 at the end of the year,
what is the annual income of A?
Solution
Income = 4 : 3, Expenses = 3 : 2
Savings 600 each
A‟s income = 4x, expenses = 3x,
savings = x i.e 600

Income = 4*600 : 3*600
A : B = 2400 : 1800
A income = 2400
Problem - 8
The property of a man was divided among his
wife, son and daughter according to his will as
follows. Wife‟s hare is equal to 6/7th of son‟s
share and daughter share is equal of 4/7th of
Son‟s. If the son and daughter together receives
Rs.1,02,300. How much does his wife get?
Solution
Let the Son‟s share be x.
Daughter‟s share = x*4/7 = 4x/7
Wife‟s share = x* 6/7 = 6x/7
X + 4x/7 = 1,02,300
7x + 4x = 1,02,300
X = 1,02,300 /11 = 65,100
Wife Share = 65,100 *6/7 = Rs. 55, 800
Problem - 9
A pot containing 81 litres of pure milk of the
milk 1/3 is replaced by the same amount of
water. Again 1/3 of the mixture is replaced by
the same amount of water. Find the ratio of milk
to water in the new mixture?
Solution
                      Milk   : Water
Initial     =         81     : 0
1/3 removed =         54     : 27
1/3 mixture =         36      : 45

Ratio of Milk and Water = 4 : 5
Problem - 10
729 ml of mixture contains milk and water are in
the ratio 7 : 2. How much more water is to be
added to get a new mixture containing milk and
water in the ratio of 7 : 3.
Solution
Water = 729 * 2/9 = 162

          Ratio             Water
             2                162
             3                 x
2x = 3*162/2 = 243
243 – 161 = 81 ml water is to be added
Problem - 11
Price of a scooter and a television set are in the
ratio 3 : 2. If the scooter costs Rs.600 more than
the television set, then find the price of
television?
Solution
Diff. in ratio = 3 – 2 = 1
1 ratio is 600 means, the television cost is 2 ratio
  so, cost of television = 1200
Problem - 12
The annual income and expenditure of man and
his wife are in the ratio of 5:3 and 3:1
respectively, if they decide to save equally and
find their balance is 4000. Find their income at
the end of the year?
Solution
Man and Wife income = 5 : 3 = 2 (diff)
Man and Wife Expenses = 3 : 1 = 2 (diff)
so, both of them are saving ratio of 2
Total saving of Man and Women = 4000,
 individual saving 2000

So, Man income = 5000 and Women income =
  3000
Problem - 13
In a class room, ¾ of the boys are above 160 cm
in height and they are in 18 number. Also out of
the total strength, the boys are only 2/3 and the
rest are girls. Find the total number of girls in a
class?
Solution
¾ of the boys in 18 numbers means, ¼ of the
  boys = 6
Total number of boys = 18+6 = 24
Ratio           Number
2/3                24
1/3                 x
2/3*x = 24*1/3
x = 24/2 = 12 Girls
Problem - 14

Rs. 770 was divided among A, B and C such that

A receives 2/ 9th of what B and C together

receive. Find A‟s share?
Solution
A = 2/9 (B+C)
B+C =9A/2
A+B+C = 770
A + 9A/2 = 770
11A = 770*2
A = 140
Problem - 15
A sporting goods store ordered an equal
number of white and yellow balls. The tennis
ball company delivered 45 extra white balls
making the ratio of white balls to yellow balls
1/5 : 1/6. How many white tennis balls did the
store originally order for?
Solution
Let the number of yellow balls be x

           (x + 45) : x = 1/5 : 1/6

Solving the above equation,

The number of white balls originally ordered

would be = 225 balls
Alligation and Mixture
Alligation and Mixture
Alligation : It is the rule that enables us to find the ratio
  in which two or more ingredients at the given price
  must be mixed to produce a mixture of a desired
  price.
(Quantity of cheaper / Quantity of costlier)
   (C.P. of costlier) – (Mean price)
= --------------------------------------
   (Mean price) – (C.P. of cheaper)
Alligation or Mixture
   Cost of Cheaper                Cost of costlier
     c                                d




                Cost of Mixture
                        m


d-m                                       m-c

(Cheaper quantity) : (Costlier quantity) = (d – m) : (m – c)
Problem -1
Three glasses of size 3 lit, 4 lit and 5 lit contain
mixture of milk and water in the ratio of 2:3, 3:7
and 4:11 respectively. The content of all the
three glasses are poured into a single vessel.
Find the ratio of milk and water in the resulting
mixture.
Solution
1st Vessel
                         = Milk = 3*2/5 = 6/5
                         = Water = 3*3/5 = 9/5

2nd Vessel:
                         = Milk = 4*3/10 = 12/10
                         = Water = 4*7/10 = 28/10
3rd Vessel:
                           = Milk = 5*4/15 = 20/15
                           = Water = 5*11/15 = 55/15
Milk : Water = 6/5 +12/10 + 20/15 : 9/5 + 28/10 + 55/15
               = 18/15 + 18/15 +20/15 : 27/15 + 42/15+55/15
               = 56 : 124 (or) 14:31
Problem - 2
How many kg of tea worth Rs. 25 per kg must be
blended with 30 kg tea worth Rs. 30 per kg, so
that by selling the blended variety at Rs.30 per
kg there should be a gain of 10%?
Solution
30*100/110 = 300/11
25                        30
         300/11

30/11                 25/11
30 :          25
6    :        5
36 :          30kg
Problem - 3
A man buys cows for Rs. 1350 and sells one so
as to lose 6% and the other so as to gain 7.5%
and on the whole he neither gains nor loses. How
much does each cow cost?
Solution
  6                 7.5
          0
  7.5               6
   15               12
   5      :     4
1350*5/9 = 750
1350 *4/9 = 600
Problem - 4
There are 65 students in a class, 39 rupees are
distributed among them so that each boy gets
80p and each girl gets 30p. Find the number of
boys and girls in a class.
Solution
Girls                Boys
30                   80
          60
20                   30
2    :         3
65*2/5 = 26
65*3/5 = 39
Problem - 5
A person covers a distance 100 kms in 10 hr
Partly by walking at 7 km per hour and rest by
running at 12 km per hour. Find the distance
covered in each part.
Solution
Speed = Distance / Time = 100 / 10 = 10
    7                          12
                   10
    2              :            3
Time taken in 7 km/hr = 10 * 2/5 = 4
4*7 = 28 km
Time taken in 12 km/hours = 10*3/5 = 6
12*6 = 72 km
Problem - 6

A merchant has 100 kg of salt, part of which
he sells at 7% profit and the rest at 17% profit.
He gains 10% on the whole. Find the quantity
sold at 17% profit?
Solution

         7                 17

                   10

       (17-10)             (10-7)
             7        :        3
The quantity of 2nd kind = 3/10 of 100kg
                         = 30kg
Problem - 7
In what ratio two varieties of tea one costing
Rs. 27 per kg and the other costing Rs. 32 per
kg should be blended to produce a blended
variety of tea worth Rs. 30 per kg. How much
should be the quantity of second variety of tea,
if the first variety is 60 kg?
Solution
                 27        32
                      30
                2          3
Quantity of cheaper tea    =    2
Quantity of superior tea        3

Quantity of cheaper tea =2*x/5 = 60 , x=150
Quantity of superior tea = 3 * 150/5 = 90 kg
Problem - 8
A 3-gallon mixture contains one part of S and

two parts of R. In order to change it to mixture

containing 25% S how much R should be

added?
Solution

                 R    :    S
                 2    :    1
                 75% :     25%
                 3    :    1
1 gallon of R should be added.
Problem - 9

Three types of tea A,B,C costs Rs. 95/kg, Rs.
100/kg. and Rs 70/kg respectively. How many
kg of each should be blended to produce 100 kg
of mixture worth Rs.90/kg given that the
quantities of B and C are equal?
Solution
     B+C/2      A
         85           95
                90
            5       5
Ratio is 1:1 so A = 50 ,    B + C = 50

The quantity would be 50 : 25 : 25
Problem - 10
In what proportion water must be added to

spirit to gain 20% by selling it at the cost price?
Solution
Profit%=20%
Let C.P =S.P= Rs.10 Then   CP=100/(100+P%)SP =25/3

             0               10
                    25/3
             5/3             25/3

The ratio is 1: 5
Problem - 11

In an examination out of 480 students 85% of
the girls and 70% of the boys passed. How
many boys appeared in the examination if total
pass percentage was 75%
Solution
Solution:
70                85

            75
10                     5

Number of Boys = 480 * 10/15
Number of Boys = 320
Problem - 12
A painter mixes blue paint with white paint so
that the mixture contains 10% blue paint. In a
mixture of 40 litre paint how many litre of blue
paint should be added, so that the mixture
contains 20% of blue paint?
Solution
Quantity of blue paint in the mixture = 10% of
  40
40*10/100 = 4
40 – 4 = 36 litre
Let x litre blur paint can be mixed
4+x/30 = 20/80 = 4+x = 9
x=5
Problem - 13
From a 100 litre mixture containing water and
milk equal proportion, 10 litres of mixture is
replaced by 10 litres of water in succession
twice. At the end, what is the ratio of milk and
water?
Solution
            Milk              Water
10 lit(1st) 50       :         50
            45       :         45
            45       :         55
2nd 10 lit 40.5      :         49.5
Add Water 40.5       :         59.5
             81      :         119
Problem - 14
In a mixture of 400 gms, 80% is copper, sliver is
20%. How much copper is to be added, so that
the new mixture has 84% copper?
Solution
  400*80/100 = 320 Copper
  400*20/100 = 80 Sliver
Percen     Mixture
 80          320
 84           x
= 320*84/80 = 336
(320+x) = (400+x) 84/100
320+x = 400+84/100 + 84x/100
16x/100 = 336 – 320; 16x/100 = 16; x = 100
Problem - 15
A jar full of whisky contains 50% alcohol. A part
of this whisky is replaced by another containing
30% alcohol and now the percentage of alcohol
was found to be 35%. Find the quantity of
whisky replaced?
Solution
     50                30
                 35
      5          :     15
5 : 15 = 1 : 3
Replaced = 3/4
Partnership
Type - 1
A invest = 10000
B invest = 15000
Profit = 5000
Find their Individual Share ?
A : B = 10000 : 15000 = 2 : 3
A‟s Share = 5000*2/5 = 2000
B‟s Share = 5000*3/5 = 3000
This is a first and basic step for any Partnership
  Problem.
Type - 2
A invest = 5000,
After 3 months B joined A, with an investment
  of 3000
Profit at the end of the year = 3500
Find their Share ?
Any thing happen after a month, like a person
  joining a business, or withdraw from
  business or withdraw some amount means
  given amount is for month.
                                     Cont…
Type - 2

A : B = 5000 : 3000 = 5*12 : 3*9 = 60 : 27 = 20 : 9

A‟s share = 3500*20/29 = 2413.7

B‟s Share = 3500*9/29 = 1086.3
Type - 3
A invest 5000
B invest 6000
After 3 months A withdraw amount 1000, after 5
  months a withdraw amount 1000 again.
Profit at the end of the Year = 5000
Find their Share ?
A = 5*3 + 4*5 + 3*4 = 15 +20 + 12 = 47
B = 6*12 = 72
Type - 3

A‟s share = 5000* 47/119 = 1974.8


B‟s share = 5000*72/119 = 3025.2
Type - 4
A invest twice as much as B, B invest 1/3rd of C. At the
  end of the year their Profit is 6000. Find their Share?
A = 2B
B = 1/3C
C=x
A : B : C = 2x/3 : x/3 : x
A : B : C = 2x/3 : x/3 : 3x /3
A:B:C=3:2:6
A‟s Share = 6000*3/11 = 1636
B‟s Share = 6000*2/11 = 1091
C‟s Share = 6000*6/11 = 3273
Problem - 1
A, B and C started a business in partnership by
investing Rs.12000 each. After 6 months, C left
and after 4 months D joined with his capital of
Rs.24,000. At the end of a year, a profit of
Rs.8,500 shared among all the partners. Find B‟s
share?
These are all the basic types remaining
 we will see when we solve problems.
Solution
  A :      B    :    C    :    D
 12000 : 12000 : 12000 : 24000
  1 :      1    :    1    :    2
  1*12 : 1*12 :      1*6 : 2*2
  12 :     12: 6     :    4
  6 :      6    :    3    :    2
B‟s share = 6/17*8500 = 3000
Problem - 2
A, B and C enter into partnership. A contributes
one third of the capital while B contributes as
much as A and C together contributed. If the
profit at the end of the year amounted to Rs.840.
What would be B‟s share?
Solution
A‟s share = 1/3 of the capital
A‟s share = 1/3*840 = 280
B‟s share = A + C = 280 + x
A + B + C = 840
280 + 280 + x + x = 840
560 + 2x = 840
2x = 840 – 560
X = 140
B‟s share = 280+140 = 420
Problem - 3
Akilesh and Jaga enter into a partnership.
Akilesh contributing Rs.8000 and Jaga
contributing Rs.10000. At the end of 6 months
they introduce Prakash, who contributes
Rs.6000. After the lapse of 3 years, they find that
he firm has made a profit of Rs.9660. Find
Prakash‟s share?
Solution
Akilesh :       Jaga :    Prakash
 8 :       10 :      6
   4       :    5    :    3
   4*36 :       5836 :    3*30
   144 :        180 :     90
   8       :    10 :      5
Prakash‟s share = 9660*5/23 = 2100
Problem - 4
Priya and Vijay enter into partnership. Priya
supplies whole of the capital amounting to
Rs.45,000 with the conditions that the profit are
to be equally divided and that Vijay pays Priya
interest on half of the capital of 10% p.a. but
receives Rs.120 per month for carrying on the
concern. Find their total yearly profit. When
Vijay‟s income is one half of Priya‟s income?
Solution
45,000 *1/2 = 22,500
22,500 *10//100 = 2250 (interest p.a)
Vijay receives Rs.120 per month = 120*12 = 1440
Total profit be x
Ratio of Profit sharing = 1 : 1
Priya‟s income = x/2+2250
Vijay‟s income = x/2 – 2250 +1440
1 Priya         = ½ Vijay
Priya Income = Twice of Vijay income
x/2 + 2250 = 2(x/2 – 2250 +1440)
X+4500/2 = 2(x/2 – 810)
X+4500/2 = x – 1620 = x +4500 = 2x – 3240
X = 7740
Total Profit of the year = 7740+1440 = 9,180
Problem - 5
Revathy and Shiva are partners sharing profits in
the ratio of 2:1. They admit Pooja into
partnership giving her 1/5th share in profits
which she acquires from Revathy and Shiva in
the ratio of 1:2. Calculate the new profit sharing
ratio?
Solution
Pooja gets her share of 1/5th of total share of
  Profit from Revathy and Shiva in the ratio 1 : 2
From Revathy = 1/3*1/5 = 1/15
From Shiva = 2/3*1/5 = 2/15
Total Pooja share = 1/15+2/15 = 3/15 = 1/15
Revathy share = 2/3 – 1/15 = 9/15
Shiva share = 1/3 – 2/15 = 3/15
Shares = Revathy : Shiva : Pooja = 3 : 1 : 1
Problem - 6
A and B started a partnership business investing
some amount in the ratio of 3 : 5. C joined them
after six months with an amount equal to that of
B. In what proportion should the profit at the end
of 1 year be distributed among A, B and C?
Solution
Let the investment,
  3 :      5    :   5
  3*12: 5*12 : 5*6
 36 :      60 :     30
  6 :      10 :     5
Problem - 7
If 4(A‟s capital) = 6(B‟s capital) = 10 (C‟s
capital) then out of a profit of rs.4650. Find C‟s
share?
Solution
Let the unknown value be x
x/4 : x/6 :      x/10
15x/60     : 10x/60 :6x/60
15 : 10 :        6
C‟s share = 6/31*4650 = Rs. 900
Problem - 8
A, B, C subscribe Rs.50,000 fro business. A
subscribes Rs.4000 more than B and B Rs.5000
more than C. Out of total profit of Rs.35,000.
Find A‟s share?
Solution
C = x,
B = x + 5000
A = x+5000+4000 = x + 9000
x +x+5000 +x+9000 = 50000
3x+14000 = 50000
3x = 50000 – 14000
3x = 36000,
x = 12000
C      :    B     :    A
12000 : 17000 : 21000
A = 35000*21/50 = 14,700
Problem - 9
A and B are partners in a business, A contributes
¼ of he capital for 15 months and B received 2/3
of the profit. For how long B‟s money was used?
Solution
B = 2/3
A = 1/3
A : B = 1/3 : 2/3 = 1 : 2
Investment
1/4x+15 : 3/4x*y
15x/4 : 3xy/4
15x/4 : 3xy/4 : : 1 : 2
30x/4 = 3xy/4
Y = 30x/4 * 4/3x = 10 months
Problem - 10
A, B and C invests Rs.4,000, Rs.5,000 and
Rs.6,000 respectively in a business and A gets
25% of profit for managing the business and the
rest of the profit is divided by A, B and C in
proportion to their investment. If in a year, A
gets Rs.200 less than B and C together, what was
the total profit for the year?
Solution
Total Profit = 100
25% for managing the business = 100 – 25 = 75%
A       :      B      :      C
 4000 : 5000 : 6000
  4     : 5 : 6
4x : 5x : 6x = 25x
100*15x/75 = 20x
A gets 4x + 25% of 20x
   = 4x + 20x *25/100 = 9x
B = 5x, C = 6x
(5x + 6x) – 9x = 200
11x – 9x = 200
2x = 200; x = 100
Total Profit 20x = 20*100 = 2000
Problem - 11
A and B entered into partnership with capitals in
the ratio of 4 : 5. After 3 months, A withdraw ¼
of his capital and B withdraw 1/5 of his capital.
The gain at the end of 10 months was Rs.760.
Find the share of B?
Solution
A     :      B
4     :      5
4000 : 5000
A‟s share = 4000*1/4 = 4000 – 1000 = 3000
B‟s share = 5000*1/5 = 5000 – 1000 = 4000
A            :          B
 3*4+3*7 : 5*3 +4*7
 12 + 21     :    15+28
  33         :      43
60*43/76 = 430
Problem - 12
Rs. 1290 is divided between A, B and C. So, that
A‟s share is 1 ½ times B‟s and B‟s share is 1 ¾
times C. What is C‟s share?
Solution
A    :    B = 1 ½ : 1 = 3/2 : 1 = 3 : 2
B    :    C = 1 ¾ : 1 = 7/4 : 1 = 7 : 4
A :B :C =3*7(A) : 2*7(B) : 7*2(B) : 4*2(C)
       = 21 : 14 : 8
B = 1290*8/43 = Rs.240
Problem - 13
A man starts a business with a capital of
Rs.90000 and employs an assistant. From the
yearly profit he keeps an amount equal to 4 ½ of
his capital and pay 35% of the remainder of the
profits. Find how much the assistant receives in a
year, in which profit is Rs.30,000.
Solution
Investment = 90,000
4 ½ of investment = 9/2/100*90000 = Rs.4050
Profit = 30,000 – 4050 = 25,950
35/100*25,950 =9082.50
Problem - 14
A and B invest in a business in the ratio 3 : 2. If
5% of the total profit goes to charity and A‟s
share is Rs. 855, what is the total profit %?
Solution
Let the total profit be Rs. 100
After paying charity A‟s share = 3/5 *95 = 57
If A‟s share is Rs. 57, the total profit is 100
If A‟s share is Rs. 855, the total profit is
            100 * 855/57 = Rs. 1500
The total profit = Rs. 1500
Problem - 15
A,B,C entered into a partnership by making an
investment in the ratio of 3 : 5 : 7. After a year
C invested another Rs. 337600 while A withdrew
Rs. 45600. The ratio of investments then
changed into 24: 59 : 167. How much did A
invest initially?
Solution
Solution:
Let the investments of A, B, and C be 3x, 5x, 7x
(3x – 45600) : 5x : (7x + 337600) = 24 : 59 : 167
  (3x – 45600)/5x = 24/59
  x = 47200
Initial investment of A = 47200 * 3 = Rs. 141600
Problems on Age
Problem - 1


The age of the Father is 4 times the age of his
Son. If 5 years ago, Father‟s age was 7 times the
age of his Son, what is the Father‟s present age?
Solution
F = 4S
F - 5 = 7(S - 5)
4S – 5 = 7S – 35
3S = 30
S = 10
Father‟s age = 4* 10 = 40 years
Problem - 2


The age of Mr. Gupta is four times the age of his
Son. After Ten years, the age of Mr. Gupta will
be only twice the age of his Son. Find the present
age of Mr. Gupta‟s Son.
Solution
G = 4S
G + 10 = 2 ( S + 10)
4S + 10 = 2S + 20
2S = 10
S=5
Son‟s Age = 5 years
Problem - 3

10 years ago Anu‟s mother was 4 times older
than her daughter. After 10 years, the mother
will be twice as old as her daughter. Find the
present age of Anu.
Solution
Ten years before:
M – 10 = 4(A – 10 )
M – 10 = 4A – 40
M = 4A – 40 + 10
M = 4A – 30
Ten Years After:
M + 10 = 2(A + 10)
M + 10 = 2A + 20
M = 2A + 20 – 10
M = 2A + 10
4A – 30 = 2A + 10
2A = 10 + 30
2A = 40: Anu‟s Age = 20
Problem – 4


The sum of the ages of A and B is 42 years. 3
years back, the ages of A was 5 times the age of
B. Find the difference between the present ages
of A and B?
Solution
A + B = 42
A = 42 – B
A – 3 = 5 ( B – 3)
A – 3 = 5B – 15
42 – B – 3 = 5B – 15
42 – 3 + 15 = 5B + B
54 = 6B
B = 54 /6 = 9
A = 42 – B; A = 42 – 9 = 33
Difference in their ages = 33 – 9 = 24 Years
Problem - 5

The sum of the ages of a son and father is 56
years. After 4 years, the age of the father will be
3 times that of the son. Find their respective
ages?
Solution
F + S= 56
S = 56 – F
F + 4 = 3 (S + 4)
F + 4 = 3 (56 – F + 4)
F + 4 = 168 – 3F + 12
4F = 168 + 12 – 4
4F = 176 ; F = 44
S = 56 – F ; S = 56 – 44 = 12
Father Age = 44; Son Age = 12
Problem – 6


The ratio of the ages of father and son at present
is 6:1. After 5 years, the ratio will become 7:2.
Find the Present age of the son.
Solution
6x + 5/x + 5 = 7/2
12x + 10 = 7x + 35
12x – 7x = 35 – 10
5x = 25
x = 25 / 5
x = 5 years
Son age = 1* 5 = 5 years
Problem - 7


The ages of Ram and Shyam differ by 16 years.
Six years ago, Shyam‟s age was thrice as that of
Ram‟s. Find their present ages?
Solution
S = R + 16
S – 6 = 3(R – 6)
S – 6 = 3R – 18
R + 16 – 6 = 3R – 18
R + 10 = 3R – 18
2R = 28 ; R = 14
Shyam‟s Age = 14 + 16 = 30.
Problem - 8


A man‟s age is 125% of what it was 10 years
ago, 83 1/3% of what it will be after 10 years.
What is his present age?
Solution
Let the age be x
125% of (x – 10) = 83 1/3 % of (x +10)
125/100 * x – 10 = 250/ 300 * x +10
5/4 x – 10 = 5/6 x – 10
5x / 4 – 5x / 6 = 50/6 + 50/4
5x /12 = 250/12
5x = 250 ; x = 50 years
Problem - 9

3 years ago, the average age of a family of 5
members was 17. A baby having born, the
average age of the family is the same today.
What is the age of the child?
Solution
Average age of 5 members = 17
Total age of 5 members = 17*5 = 85
3 years later, the age of 5 members will be
= 85 + 15 = 100
100 + x / 6 = 17
100 + x = 17*6
100 + x = 102
x = 102 – 100 = 2 years
Problem - 10
The sum of the age of father and his son is 100
years now. 5 years ago their ages were in the
ratio of 2 : 1. The ratio of the ages of father and
his son after 10 years will be?
Solution
F + S = 100
5 years ago 2 : 1
5 years ago
F + S = 100 – 10 = 90
90*2/3 = 60 : 30
Present age = 65 : 35
10 years ago = 75 : 45
=5:3
Problem - 11
Six years ago, Sushil‟s age was triple the age of
Snehal. Six years later, Sushil‟s age will be 5/3
of the age of Snehal. What is the present age of
Snehal?
Solution
Six years ago,
Snehal = x; Sushil = 3x
Six years later,
3x + 6+6 = 5/3(x+6+6)
9x +36 = 5x+60
4x = 60 – 36
X=6
Present Age of Snehal = 6+6 = 12 years
Problem - 12

Susan got married 6 years ago. Today her age is
1¼ times that at the time of her marriage. Her
son is 1/6 as old as she today. What is the age of
her son?
Solution
6 years ago Susan got married.
So her son‟s age will be less than 6 years.
Let as consider, her son‟s age is 5 years.
Susan‟s Age is 5*6 = 30 yrs, since the son is 1/6th of
  Susan‟s age.
6 years ago her age must have been 24 yrs
24*1 ¼ = 24*5/4 = 30 yrs
As it satisfies the conditions her son‟s age is 5 years
Problem - 13
My brother is 3 years elder to me. My father was
28 years of age when my sister was born, while
my mother was 26 years of age, when I was
born. If my sister was 4 years of age when my
brother was born, then, what was the age of my
father and mother respectively when my brother
was born?
Solution
My brother was born 3 years before I was born
  and 4 years after my sister was born
Father‟s age when brother was born
                      = 28 + 4 = 32 years
Mother‟s age when brother was born
                      = 26 – 3 = 23 years
Problem - 14

If 6 years are subtracted from the present age of
Gagan and the reminder is divided by 18, then
the present age of his grandson Aunp is obtained.
If Anup is 2 years younger to Madan whose age
is 5 years, then what is Gagan‟s present age?
Solution
Anup‟s age = 5 – 2 = 3 years
Let Gagan‟s age be x
= x – 6 / 18 = 3
x – 6 = 3*18 ; x – 6 = 54
x = 54 + 6
Gagan‟s age = 60
Problem - 15

Ramu‟s grandfather says, “ Ram, I am now 30
years older than your father. 15 years ago, I was
2½ times as old as your father”. How old is the
grandfather now?
Solution
Let the father‟s age be x.
Grandfather‟s age will be 30 + x
15 years ago,
X + 30 – 15 = 5/2 (x – 15)
X + 15 = 5/2 (x – 15)
2x + 30 = 5x – 75
105 = 3x
X = 105 / 3 = 35
Grandfather‟s age = 35 + 30 = 65
Average
Average
Average   =       Sum of Quantities
                 Number of Quantities
Sum of quantities
          =     Average*Number of Quantities.

Number of quantities
          =    Sum of Quantities
                   Average
Problem - 1

The average age of a class of 22 students is 21
years. The average increases by 1 when the
teacher‟s age is also included. What is the age of
the teacher?
Solution
Total age of the students be x
x/22 = 21; x = 21*22= 462
Teacher‟s age is also included
x/23 = 22; x = 22*23 = 506
Total age of 23 people – Total age of 22 people
will be the age of teacher
506 – 462 = 44 years
The age of teacher = 44
Problem - 2

The average of 7 numbers is 25. The average of
first three of them is 20 while the last three is 28.
What must be the remaining number?
Solution
Average of 7 numbers = 25,
Sum of 7 numbers = 25* 7 = 175
Avg. of first three numbers = 20, 20* 3 = 60
Avg. of last three numbers = 28, 28*3 = 84
The 4th number = 175 – (60+84) = 175 – 144
= 31
Problem - 3

The average age of a team of 10 people remains
the same as it was 3 years ago, when a young
person replaces one of the member. How much
younger was he than the person whose place he
took?
Solution
Let Average be x
10 members‟ Average = 10x
Average of 10 members (including new one) is
same as it was 3 yrs ago.
Now 10*3 = 30 years have increased, so a person
of 30 years should have replaced to keep the
average as same.
Problem - 4

The average age of a couple was 26 years at that
time of their marriage. After 11 years of marriage
the average age of the family with 3 children
become 19 years. What is the average age of the
Children?
Solution

Average of parents ages is 26, sum= 26*2 = 52
Parents age after 11 years = 52 +22 = 74
Average age of Family = 19, Sum = 19*5 = 95
Sum of family‟s age – Sum of parents‟ age
= 95 – 74 = 21
Sum of the ages of 3 children = 21,
Average Age = 21/3 = 7 yrs
Problem - 5

9 members went to a hotel for taking meals.
Eight of them spent Rs. 12 each on their meals
and the ninth person spent Rs. 8 more than the
average expenditure of all the nine. What was
the total money spent by them?
Solution
Average = x/9
Amount Spent by 8 members = 12 * 8 = 96
96 + x/9 + 8 = x
104 = x – x/9
104 = 8x/9
8x = 104 *9 = 936
x = 936/8 = 117
Problem - 7

A batsman makes a score of 87 runs in the 17th
inning and thus increases his average by 3. Find
his average after 17th innings?
Solution
17th innings avg. = x, Runs = 17x
16th innings avg. = x -3, Runs = 16 (x -3)
16 (x-3) + 87 = 17x
16x – 48 +87 = 17x
X = 39
Problem - 7
There are 24 students in a class. One of them,
who was 18 yrs old left the class and his place
was filled up by the newcomer. If the average of
the class thereby was lowered by one month,
what is the age of the newcomer?
Solution
Average reduced by 1 month,
24 * 1 = 2 years
So, the newcomer‟s age is 18 -2 = 16 years
Problem - 8
The average of marks in mathematics for 5
students was found to be 50. Later, it was
discovered that in the case of one student the
mark 48 was misread as 84. What is the correct
average?
Solution
Difference = 84 – 48 = 36
36 /5 = 7.2 (Increased)
The corrected average = 50 – 7.2 = 42.8
Problem - 9
The average salary of all the workers in a factory
is Rs. 8000. The average salary of 7 technicians
is Rs. 12000 and the average salary of the rest is
Rs. 6000. What is the total number of workers in
the factory?
Solution
Members              Avg.
  7                 12000
  X                  6000
  6x = 7*12
  X = 7812/6 = 14
  Total no. of workers = 7 + 14 = 21
Problem - 10
Average salary of all the 50 employees including
5 officers of the company is Rs. 850. If the
average salary of the officers is 2500, find the
average salary of the remaining staff of the
company.
Solution
x/50 = 850; x = 42,500
5 officers‟ salary = 2500*5 = 12500
50 – 5 members = 42500 – 12500
45 members = 30000
Avg. salary of 45 members = 30000/45
                            = 667(App)
Problem - 11

Find the average of 8 consecutive odd numbers
21,23,25,27,29,31,33,35
Solution

1st number + last Number /2
= 21 + 35 /2 = 28
Problem - 12

A train covers 50% of the journey at 30 km/hr,
25% of the journey at 25 km/hr, and the
remaining at 20 km/hr. Find the average speed of
the train during entire journey.
Solution
Total Journey = 100 km
S = Distance / Time = 100 / 5/3 + 1/1 + 5/4
= 100 * 12 /20+12+15
= 1200/47 = 25 25/47 km/hr
Problem - 13

The average of 10 numbers is 7. What will be the
new average if each number is multiplied by 8?
Solution
If numbers are multiplied by 8,
Average also to be multiplied by 8
= 7*8 = 56
{or}
x/10 = 7
x = 10*7 = 70
= 70* 5 = 560 /10 = 56
Problem - 14
The mean marks of 10 boys in a class is 70%
whereas the mean marks of 15 girls is 60%.
What is the mean marks of all 25 students?
Solution
Boys = x/10 = 70 = 700
Girls = x/15 = 60 = 900
10 + 15 = 700 + 900
25 = 1600
1600/25 = 64%
Problem - 15
Of the three numbers the first is twice the second
and the second is thrice the third. If the average
of the three numbers is 10, what are the
numbers?
Solution
A = 2x
B=x
C = x/3
2x + x + x/3/3 = 10
6x + 3x + x /9 = 10
6x + 3x + x = 90
10x = 90 ; x = 9.
A = 18, B = 9, C = 3
Percentage
Percentage

• By a certain Percent, we mean that many

hundredths.

• Thus, x Percent means x hundredths, written

as x%
Percentage
•Finding out of Hundred.
If Length is increased by X% and Breadth is
decreased by Y% What is the percentage
Increase or Decrease in Area of the rectangle?
       Formula: X+Y+ XY/100 %
       Decrease 20% means -20
Problem -1

When 75% of the Number is added to 75%, the
result is the same number. What is the number?
Solution
Percentage Number
     75        x+75
     100        x
100x + 7500 = 75x
25x = 7500
x = 300
Problem - 2
A tank is full of milk. Half of the milk is sold
and the tank is filled with water. Again half of
the mixture is sold and the tank is filled with
water. This operation is repeated thrice. Find the
percentage of milk in the tank after the third
operation?
Solution
Milk                  Water
100                     0
50                     50(1st)
 25                   75 (2nd)
 12.5                 87.5 (3rd)
After 3 operation Milk 12.5%
Problem 3

A large water-melon weighs 20kg with 96% of
its weight being water. It is allowed to stand in
the sun and some of the water evaporates so that
now, only 95% of its weight is water. What will
be its reduced weight?
Solution
20 *96/100=19.2kg of water
Let the evaporated water be x
19.2-x=95%(20-x)
19.2-x=95(20-x)/100
1920-100x=1900-95x
5x=20 ;x=4
20-4=16kg.
Problem 4
The population of a city is 155625. For
every1000 men, there are 1075 women. If 40%
of men and 24% of women be literate, then what
is the percentage of literate people in the city?
Solution
Ratio of men and women=1000:1075=40:43
Number of men=40*155625/83=75000
Number of women=155625-7500=80625
Number of literate men=75000*40/100=3000
Number of literate women
=80625*24/100=19350
Literate people =30000+19350=49350
Percentage of literate people
=49350/155625*100=2632/83=31 59/83%
Problem 5

300 grams of sugar solution has 40% sugar in it.
How much sugar should be added to make it
50% in the solution?
Solution
Grams                  Sugar
300                     40%
X                       50%
50x = 40*300
x = 40*300/50 = 240
300 – 240 =60 Kg
Problem - 6

A man lost 12½% of his money and after
spending 70% of the remainder, he has Rs. 210
left. How much did the man have at first?
Solution
Let the amount be 100
Then, 100.00 – 12.50 = 87.50

70% of 87.50 = 87.50 *70/100 =61.25

The remaining amount will be Rs. 26.25
  Initial   Final
  100       26.25
  X         210
26.25x = 21000; x = 21000/26.25 = 800
Problem - 7
During one year the population of a town
increases by 10% and during next year it
diminished by 10%. If at the end of the second
year, the population was 89,100, what was the
Population at the beginning of first year?
Solution
Let the population be 100
1st Year = 100 + 10 = 110
2nd Year = 110 * 10/100 = 110 -11 = 99
Percentage            Population
99                     89100
100                        x
99x = 89100*100;
x = 8910000/99 = 90000
Problem - 8

When a number is first increased by 20% and
then again 20% by what percent should the
increase number be reduced to get back the
original number?
Solution
Let the number be 100
20% increase = 100*20/100 = 20
New Value = 120
Again increase by 20% = 120*20/100 = 24
New value = 144
Increased amount = 44/144*100 = 30 5/9%
Problem - 9
The number of students studying Arts,
Commerce and Science in an institute were in
the ratio 6 : 5 : 3 respectively. If the number of
students in Arts, Commerce and science were
increased by 10%, 30% and 15% respectively,
what was the new ratio between number of
students in the three streams?
Solution
A:C:S
6:5:3
6x : 5x : 3x
6x*110/100 : 5x*130/100 : 3x*115/100
6x*110 : 5x*130 : 3x*115
660 : 650 : 345
132 : 130 : 69
Problem - 10

In measuring the sides of rectangle errors of 5%
and 3% in excess are made. What is the error
percent in the calculated area?
Solution

Area = xy
X = 5% Excess = 100* 5/100 = 105
Y = 3% Excess = 100*3/100 = 103
103*105/100 = 10815/100 = 108.15
Error – Actual = 108.15 – 100 = 8.15% Excess
Problem - 11

In a certain examination there were 2500
candidates. Of them 20% of them were girls and
rest were boys. If 5% of boys and 40% of girls
failed, what was the Percentage of candidates
passed?
Solution
Girls = 2500*20/100 = 500
Boys = 2500*80/100 = 2000
Students who failed were
Boys = 2000*5/100 = 100
Girls = 500*40/100 = 200
Total Failed Students = 300
Total Pass students = 2500 – 300 = 2200
Pass Percentage = 2200/2500*100 = 88%
Problem - 12

A person saves every year 20% of his income. If
his income increases every year by 10% then his
saving increases by?
Solution
Every year saving, if the income is Rs. 100
     = 100 *20/100 =Rs. 20
Salary increases = 110*20/100 = 22
Percentage increase (Savings) = 2/20*100 = 10%
Problem - 13
On a test containing 150 questions carrying 1
mark each, meena answered 80% of the first
answers correctly. What percent of the other 75
questions does she need to answer correctly to
score 60% on the entire exam?
Solution
Required correct answer = 150*60/100 = 90
  Questions need to be correct.
80% of 75 questions = 60 q answered correctly.
Remaining 30 questions need be correct out of 75
= 30/75*100 = 40
Problem - 14

A boy after giving away 80% of his pocket
money to one companion and 6% of the
remainder to another has 47 paise left with him.
How much pocket money did the boy have in the
beginning?
Solution
Let the amount be 100
To the first companion = 100*80/100 = 80
Remaining = 100 – 80 = 20
To the 2nd person = 20*6/100 = 1.20
The remaining = Rs.18.80 or 1880 paise
Initial             Final
100                 1880
X                    47
1880x = 47*100
x = 4700/1880 = 2.5
Problem - 15
The length of a rectangle is increased by 10%
and breath decreased by 10%. Then the area of
the new rectangle?
Solution
I – D – I*D /100
10 -10 – 10*10/100
0 – 1 = -1
Decrease by 1%
Profit and Loss
Profit and Loss
• Gain =(S.P.)-(C.P.)
• Loss =(C.P.)-(S.P.)
• Loss or gain is always reckoned on C.P.
• Gain% = [(Gain*100)/C.P.]
• Loss% = [(Loss*100)/C.P.]
• S.P. = ((100 + Gain%)/100)C.P.
• S.P. = ((100 – Loss%)/100)C.P.
Problem - 1
A trade man allows two successive discount of
20% and 10%. If he gets Rs.108 for an article.
What was its marked price?
Solution
I1 + I2 – I1*I2/100
20 + 10 – 20*10 /100
= 28%
Discount = 28%, 72 Percent Cost is 108
Then 100percent cost = 72     108
                      100       x
100*108/72 = 150
Problem - 2
A trade man bought 500 metres of electric wire
at 75 paise per metre. He sold 60% of it at profit
of 8%. At what gain percent should he sell the
remainder so tas to gain 12% on the whole
Solution
500* 60/100 = 300
8                 X
       12
300          200
300 : 200 = 6 : 4
8                 18
       12
6                 4
Remainder at 18% Profit
Problem - 3
A man purchased a box full of pencils at the rate
of 7 for Rs. 9 and sold all of them at the rate of 8
for Rs. 11. in this bargains he gains Rs. 10. How
many pencils did the box contains.
Solution
LCM = 7 and 8 = 56
56 pencil cost price = 8*9 = 72
56 Pencil selling price = 7*11
Profit = 77 – 72 = Rs. 5 for 56 pencil
Rs. 5 for 56 pencil means , for Rs. 10 the pencils
  are 112
Problem - 4
A cloth merchant decides to sell his material at
the cost price, but measures 80cm for a metre.
His gain % is?
Solution
100 – 80 = 20 cm difference
Actual = 80
20/80*100 = 25% Gain
Problem - 5
Sales of a book decrease by 2.5% when its price
is hiked by 5%. What is the effect on sales?
Solution
Let the sales be 100 – 2.5 = 97.5
Profit = 100+5 = 105
Sales                  Profit
97.5                   105
100                    X
100x = 97.5*105
x = 97.5*105/100 = 102.375
100 – 102.375 = 2.375 = 2.4 profit (app)
Problem - 6
A dealer buys a table listed at Rs.1500 and gets
successive discount of 20% and 10%. He spends
Rs. 20 on transportation and sells it at a profit of
10%. Find the selling price of the table.
Solution
Discount = 20+10 – 20*10/100 = 28%
Actual price = 100 – 28 = 72
100                      1500
 72                      x
72*1500/100 = 1080
Transport = 1080 +20 = 1100
100                      1100
 110                       x
1100*110/100 = 1210
Problem - 7
A fridge is listed at Rs. 4000. due to the off
season, a shopkeeper announces a discount of
5%. What is the S.P?
Solution
= 4000*95/100 = 3800
Problem - 8
If the cost price of 9 pens is equal to the S.P of
11 pens. What is the gain or loss?
Solution
= 11 – 9 = 2
= 2/11*100 = 18 2/11% loss
Problem - 9
A machine is sold for Rs.5060 at a gain of 10%
what would have been the gain or loss percent if
it had been sold Rs.4370?
Solution
S.P = Rs.5060 = Gain = 10%
C.P = 100/110*5060 = 4600
IF S.P = Rs.4370 and C.P = Rs.4600
Loss = 230
Loss % = 230/4600 * 100 = 5% loss
Problem - 10
A person purchased two washing machines each
for Rs.9000. he sold one at a loss of 10% and
other at a gain of 10%. What is his gain or loss?
Solution
Each Rs.9000. one is 10% profit and other is
10% loss. So No profit and No loss
Problem - 11
Four percent more is gained by selling an article
for Rs.180, then by selling if for Rs.175. then its
C.P is?
Solution
Let the cost price = Rs. X
4% of x = 180 – 175 = 4x/100 = 5
4x = 500; x = 500/4 = 125
Problem - 12
An article is sold at a profit of 20%. If it had
been sold at a profit of 25%. It would have
fetched Rs.35% more. The Cost Price of the
article is?
Solution
Let C.P = Rs. X
125% of x – 120% of x = 35
5% of x =Rs.35 = x = 35*100/5 = 700
C.P = Rs. 700
Problem - 13
A reduction of 20% in the price of orange
enables a man to buy 5 oranges more for Rs. 10.
The price of an orange before reduction was,
Solution
20% Rs. 10 = Rs.2
Reduced price of 5 oranges = Rs. 2
Reduced price of 1 oranges = 40 p
Original price = 40/ 1- 0.20 = 400/8 = 50 Paise
Problem - 14
A man sells two horses for Rs.1475. The cost
price of the first is equal to the S.P of the second.
If the first is sold at 20% loss and the second at
25% gain. What is his total gain or loss? ( in
rupees)
Solution
Let cost price of 1st horse = S.P of 2nd = x
C.P of 2nd = S.P of 2nd * 100/125 = x*100/125 =
  4x/5
S.P of 1st = C.P of 1st *80/100 = x*80/100 = 4x/5
Neither loss nor gain
Problem - 15
Rekha sold a watch at a profit of 15%. Had he
bought it at 10% less and sold it for Rs. 28 less,
he would have gained 20%. Find the C.P of the
Watch.
Solution
C.P be Rs. X
First S.P = 115% of x = 23x/20 and second C.P =
  90% x = 9x/10
Second S.P = 120% of 9x/10 = 120/100 * 9x/10
  = 27x/25
Given 23x/20 – 27x/25 = 28 = 115x – 108x/100
  = 28
7x/100 = 28 = x = 28*100/7 = 400
C.P = Rs.400
Probability
Probability
• Probability:
    P(є) = n(є) / n(s)
• (Addition theorem on probability:
    n(AUB) = n(A) + n(B) - n(A B)
• Mutually Exclusive:
    P(AUB) = P(A) + P(B)
• Independent Events:
    P(A B) = P(A) * P(B)
Problem - 1
Four cards are drawn at random from a pack of
52 playing cards. Find the probability of getting
all face cards?
Solution
n(E) = 52C4
n(S) = 12C4 = 12C4/52C4
Problem - 2
Four persons are to be chosen at random from a
group of 3 men, 2 women and 4 children. Find
the probability of selecting 1 man, 1 woman or 2
children?
Solution
Total 3 M + 2 W + 4 C = 9 C 4 = 126
n (E) = 3C1 * 2C1 * 4C2 = 36
36/126 = 2/7
Problem - 3
A word consists of 9 letters, 5 consonants and 4
vowels. Three letters are chosen at random.
What is the probability that more than one
vowels will be selected?
Solution
n(E) = 9C3 = 84
More than one Vowels. So,
2V +1C or 3 V
4C2 *5C1 + 4C3 = 34
= 34/84 = 17/42
Problem - 4
A bag contains 10 mangoes out of which 4 are
rotten. Two mangoes are taken out together. If
one of them was found to be good, then what is
the probability that the other one is also good?
Solution
10 mangoes – 4 are rotten = 6 good mangoes
Getting good mangoes = 6C1/10C1 = 6/10
Getting second mango to be good = 5/9
1st and 2nd mangoes
6/10 *5/9 = 1/3
Problem - 5
Out of 13 applicants for a job there are 5 women
and 8 men. It is desired to select 2 persons for
the job. What is the probability that at least one
of the selected person will be a woman?
Solution
n(E) = 13C2 = 78
n(S) = 1m and 1 w or 2 w
      = 8C1*5C1 + 5C2 = 50
      = 50/78 = 25/39
Problem - 6
Two cards are drawn at random from a pack of
52 cards. What is the probability that either both
are black or both are queen?
Solution
P(A) = Both are Black
P(B) = Both are Queen
P(AnB) = Both are queen and Black
P(A) = 26C2/52C2 = 325/1326
P(B) = 4C2 /52C2 = 6/1326
P(AnB) = 2C2 /52C2 = 1/1326
325/1326 + 6/1326 - 1/ 1326 = 55/221
Problem -7
A man and his wife appear in an interview for
two vacancies in the same post. The probability
of husband‟s selection is 1/7 and the probability
of wife‟s selection is 1/5. Find the probability
that only one of them is selected?
Solution
Husband‟s Selection = 1/7;
Not getting selected = 1 – 1/7 = 6/7
Wife‟s selection = 1/5;
Not getting selected = 1 – 1/5 = 4/5
Only one of them is selected =
(Husband‟s Selection + Wife Not getting selected) or
(Wife‟s selection + Husband‟s Not getting selected)
= (1/7*4/5) + 1/5*6/7) = 2/7
Problem - 8
Four persons are chosen at random from a group
of 3 men, 2 women and 4 children. What is the
chance that exactly 2 of them are children?
Solution
3 + 2 + 4 = 9C4 = 126
4 members 2(M and W) + 2(boy)
5C2 + 4C2 = 60
= 60 / 126 = 10/21
Problem - 9
Prakash can hit a target 3 times in 6 shots, Priya
can hit the target 2 times in 6 shots and Akhilesh
can hit the target 4 times in 4 shots. What is the
probability that at least 2 shots hit the target?
Solution
Prakash hitting = 3/6; not hitting = 3/6
Priya hitting = 2/6; not hitting = 4/6
Akilesh = 4/4 = 1
At least 2 shots hit target
            = 3/6*4/6 + 3/6*2/6 = ½
Problem - 10
There are two boxes A and B. A contains 3 white
balls and 5 black balls and Box B contains 4
white balls and 6 black balls. One box is taken at
random and what is the probability that the ball
picked up may be a white one?
Solution
(Box A is selected and a ball is picked up ) or
(Box B is selected and a ball is picked up)
½*3/8 + ½*4/10 = 31/80
Problem - 11
A bag contains 6 white balls and 4 black balls.
Four balls are successively drawn without
replacement. What is the probability that they are
alternately of different colour?
Solution
Suppose the balls drawn are in the order white,
black, white, black…
= 6/10 *4/9*5/8*3/7 = 360/5040
Suppose the balls drawn are in the order black,
white, black, white…
= 4/10*6/9*3/8*5/7 = 360/5040
360/5040 +360/5040 = 1/7
Problem - 12
A problem in statistics is given to four students
A, B, C and D. Their chances of solving it are
1/3, ¼, 1/5 and 1/6 respectively. What is the
probability that the problem will be solved?
Solution
A is not solving problem = 2/3,
B is not solving problem = ¾
C not solving problem = 4/5
D not solving problem = 5/6
2/3*3/4*4/5*5/6 = 1/3
All together the probability of solving the
problem = 1 -1 /3 = 2/3
Problem - 13
There are 8 questions in an examination each
having only 2 answers choices „Yes‟ or „No‟. All
the questions carry equal marks. If a student
marks his answer randomly, what is the
probability of scoring exacting 50%?
Solution
Each questions having 2 ways of answering,
1 question = 2!........ 8 question = 2!
  = 2!*2!*2!*2!*2!*2!*2!*2! = 256
To get 50%, 4 questions need to be correct,
8c4 = 8*7*6*5/1*2*3*4 = 70
   = 70/256 = 35/128
Problem - 14

A group consists of equal number of men and
women. Of them 10% of men and 45% of
women are unemployed. If a person is randomly
selected from the group find the probability for
the selected person to be an employee.
Solution

Let the number of men is 100 and women be 100
Employed men and women = (100-10)+(100-45)
                          = 145
          Probability = 145 / 200 = 29 / 40
Problem - 15
The probability of an event A occurring is 0.5
and that of B is 0.3. If A and B are mutually
exclusive events. Find the probability that
neither A nor B occurs?
Solution

It is Mutually exclusive events P(A n B)=0
Probability = 1 – ( P(A) + P (B) – P(A n B) )
            = 1 – (0.5 + 0.3 – 0)
            = 0.2
Permutation and Combination
Permutation and Combination

Permutation means Arrangement


Combination means Selection
Permutation and Combination
• Permutations:
    Each of the arrangements which can be made by
   taking some (or) all of a number of items is called
   permutations.
np = n(n-1)(n-2)…(n-r+1)=n!/(n-r)!
  r
• Combinations:
    Each of the groups or selections which can be made
   by taking some or all of a number of items is called a
   combination.
nC    = n!/(r!)(n-r)!
  r
Types
1. How many ways of Arrangement possible by
  using word SOFTWARE?
  SOFTWARE = 8!
2. How many ways of arrangement Possible by
  using word SOFTWARE, vowels should come
  together.
  SFTWR (OAE) = 6! * 3!
Types
3. How many ways of Arrangement Possible by
  using word SOFTWARE, vowels should not
  come together?
  SFTWR ( ARE)
  Not together
  = Total arrangement – Vowels together
  = 8! – (5! * 3!)
Types
4. How many ways of arrangement possible by using
   word MACHINE, so that vowels occupy only ODD
   places.
   - - - - - - - (7 places)
   MCHN (AIE) 4 Consonant and 3 vowels.
   7 places = 4 ODD places, 3 EVEN places
Vowels = 4P3 = 4!
Consonant = 4P4 = 4!
Total Number of arrangement = 4!*4!
Types
5. How many ways of arrangement possible by
  using word ARRANGEMENT
  Letter‟s Repetition = 2(A) 2(R) 2 (E) 2 (N)
  = 11!/2!*2!*2!*2!
In a given problem, any letter is repeated more
  than once that should be divided with total
  number.
Problem - 1

A committee of 5 is to be formed out of 6 gents
and 4 ladies. In how many ways this can be
done, when at least 2 ladies are included?
Solution
a. 2 ladies * 3 Gents
4C2 * 6 C3 = 120
b. 3 ladies * 2 Gents
4C3 * 6C2 = 60
c. 4 ladies * 1 Gent
4C4 *6C1 = 1*6 = 6
Total ways = 120 +60 +6 = 186
Problem - 2

It is required to seat 5 men and 4 women in a
row so that the women occupy the even places.
How many such arrangements are possible?
Solution
Total places = 9
Odd places = 5
Even places = 4
4 even places occupied by 4 women
= 4P4 = 4! = 24
5 odd places occupied by 5 men
= 5P4 = 5! = 120
Total ways = 120*24 = 2880 ways
Problem - 3

A set of 7 parallel lines is intersected by another
set of 5 parallel lines. How many parallelograms
are formed by this process?
Solution

Two parallel lines from the first set and any two
from the second set will from a parallelogram.
7C2 *5C2 = 21 * 10 = 210
Problem - 4

There are n teams participating in a football
championship. Every two teams played one
match with each other. There were 171 matches
on the whole. What is the value of n?
Solution
Total number of matches played = nC2
nC2 = 171
n(n-1)/2= 171
n2 – n – 342 = 0
(n+18) (n-19) = 0
 n = 19
Problem - 5

In an examination, a candidate has to pass in
each of the 6 subjects. In how many ways can he
fail?
Solution

6C1 + 6C2 + 6C3 + 6C4+6C5+6C6
1 + 6 + 15 + 20 + 15 + 6 = 63 ways
Problem - 6

In how many ways can a pack of 52 cards be
distributed to 4 players, 17 cards to each of 3 and
one card to the fourth player?
Solution
17 cards can be given to 1st player = 52 C17
2nd player = 35C17
3rd player = 18C17
4th player = 1
= 52C17*35C17*18C17
= 52!/17!35! * 35!/17!*18! * 18!/17!*1!
= 52!/(17!)3
Problem - 7
A foot race will be held on Saturday. How many

different arrangements of medal winners are

possible if medals will be for first, second and

third place, if there are 10 runners in the race …
Solution
n = 10
r=3
n P r = n!/(n-r)!
     = 10! / (10-3)!
     = 10! / 7!
     = 8*9*10
   = 720
Number of ways is 720.
Problem - 8

To fill a number of vacancies, an employer must
hire   3    programmers    from    among     6
applicants, and two managers from 4 applicants.
What is total number of ways in which she can
make her selection ?
Solution
It is selection so use combination formula

Programmers and managers = 6C3 * 4C2

                            = 20 * 6 = 120

     Total number of ways = 120 ways.
Problem - 9

A man has 7 friends. In how many ways can


he invite one or more of them to a party?
Solution
In this problem, the person is going to select his
friends for party, he can select one or more
person, so addition
  = 7C1 + 7C2+7C3 +7C4 +7C5 +7C6 +7C7
  = 127
   Number of ways is 127
Problem - 9

Find the number of different 8 letter words

formed from the letters of the word EQUATION

if each word is to start with a vowel
Solution
For the words beginning with a vowel, the first
letter can be any one of the 5 vowels, the
remaining 7 places can be filled by
       7P7 = 5040
The number of words = 5 * 5040 = 25200
Problem - 10

In how many different ways can the letters of the

word TRAINER be arranged so that the vowels

always come together?
Solution
A,I,E can be arranged in 3! Ways

        (5! * 3!) / 2! = 360 ways
Problem - 11
In how many different ways can the letters of the
Word DETAIL be arranged so that the vowels
may occupy only the odd positions?
Solution
___ ___ ___ ___ ___ ___
3P3 = 3! = 1*2*3 = 6
3P3 = 3! = 1*2*3 = 6
   = 6*6 = 36
Problem - 12
There are 5 red, 4 white and 3 blue marbles in a
bag. They are taken out one by one and arranged
in a row. Assuming that all the 12 marbles are
drawn, find the number of different
arrangements?
Solution
Total number of balls = 12
Of these 5 balls are of 1st type (red), 4 balls are
 the 2nd type and 3 balls are the 3rd type.
Required     number      of    arrangements       =
 12!/5!*4!*3!
 = 27720
Problem - 13
5 men and 5 women sit around a circular table,
the en and women alternatively. In how many
different ways can the seating arrangements be
made?
Solution
5 men can be arranged in a circular table in 4
  ways = 24 ways
There are 5 seats available for 5 women they can
  be arranged in 5 ways
No. of ways = 5!*4! = 2880 ways
Problem - 14

In a chess board there are 9 vertical and 9
horizontal lines. Find the number of rectangles
formed in the chess board.
Solution
Solution:



            9C2 * 9C2 = 1296
Problem - 15
In how many ways can a cricket team of 11
players be selected out of 16 players, If one
particular player is to be excluded?
Solution
Solution:
 If one particular player is to be excluded, then
  selection is to be made of 11 players out of 15.
           15C11= 15!/( 11!*4!)=1365 ways
Area and Volume
Area and Volume
Cube:

• Let each edge of the cube be of length a. then,

• Volume = a3cubic units

• Surface area= 6a2 sq.units.

• Diagonal = √3 a units.
Area and Volume
Cylinder:

• Let each of base = r and height ( or length) = h.

• Volume = πr2h

• Surface area = 2 πr h sq. units

• Total Surface Area = 2 πr ( h+ r) units.
Area and Volume
Cone:
• Let radius of base = r and height=h, then
• Slant height, l = √h2 +r2 units
• Volume = 1/3 πr2h cubic units
• Curved surface area = πr l sq.units
• Total surface area = πr (l +r)
Area and Volume
Sphere:

• Let the radius of the sphere be r. then,

• Volume = 4/3 πr3

• Surface area = 4 π r2sq.units
Area and Volume
Circle: A= π r 2
Circumference = 2 π r
Square: A= a 2
Perimeter = 4a
Rectangle: A= l x b
Perimeter= 2( l + b)
Area and Volume
Triangle:

A = 1/2*base*height

Equilateral = √3/4*(side)2

Area of the Scalene Triangle

 S = (a+b+c)/ 2

A = √ s*(s-a) * (s-b)* (s-c)
Problem - 1
A rectangular sheet of size 88 cm * 35 cm is bent
to form a cylindrical shape with height 35 cm.
What is the area of the base of the cylindrical
shape?
Solution
The circumference of the circular region = 88 cm
2 r = 88
r = 88*7/22*2 = 14 cm
Area of the base = r2 = 22/7*14*14 v= 616 cm2
Problem - 2
The radius of the base of a conical tent is 7
metres. If the slant height of the tent is 15
metres, what is the area of the canvas required to
make the tent?
Solution
R=7m
L = 15 m
Area of Canvas required = Curved Surface Area
of cone
  rl = 22/7*7*15 = 330 sq.m
Problem - 3
Three spherical balls of radius 1 cm, 2 cm and 3
cm are melted to form a single spherical ball. In
the process, the material loss was 25%. What
would be the radius of the new ball?
Solution
Vol. of sphere = 4/3 r3
Vol. of 3 small spherical balls = 4/3 ( 13+23+33)
= 4/3 (1+8+27) = 4 /3 (36) = 48
Material loss = 25%
Vol. of the single spherical ball = 48 *75/100
                              = 48 * ¾ = 36
                          V = 4/3 r3 = 36
                          r3 = 36*3/4 = 27
                          r = 3 cm
Problem - 4

A rectangular room of size 5m(l)*4m(w)*3m(h)
is to be painted. If the unit of painting is Rs. 10
per sq.m, what is the total cost of painting?
Solution
Area of 4 walls = 2h(l+b)
The area to be painted includes the 4 walls and
the top ceiling.
Area to be painted = 2h (l+b) +lb
                       = 2*3 (5+4) + 5*4
                       = 54+20 = 74 sq.m.
Total cost of painting = 74*10 = Rs.740
Problem - 5
The radius of a sphere is r units. Each of the
radius of the base and the height of a right
circular cylinder is also r units. What is the ratio
of the volume of the sphere to that of the
cylinder?
Solution
Vol. of sphere = 4/3 r3 and Vol. of Cylinder =
   r2h = r3

Required Ratio = 4/3 r3:   r3 = 4/3 : 1
                =4:3
Problem - 6
A measuring jar of internal diameter 10 cm is
partially filled with water. Four equal spherical
balls of diameter 2 cm each, are dropped in it
and they sink down in the water completely.
What will be the increase in the level of water in
the jar?
Solution
Radius of each ball = 1 cm
Vol. of 4 balls = 4* 4/3 (r)3 = 16/3 cm3
Vol. of water raised in the Jar = Vol. of 4 balls
Let h be the rise in water level, then
Area of the base *h = 16/3
 *5*5*h = 16/3
H = 16/3*25 = 16/75 cm
Problem - 7


What is the cost of planting the field in the form

of the triangle whose base is 2.8 m and height

3.2 m at the rate of Rs.100 / m2
Solution
Area of triangular field = ½ * 3.2 * 2.8 m2
                       = 4.48 m2
                  Cost = Rs.100 * 4.48
                       = Rs.448..
Problem - 8
Find the length of the longest pole that can be
placed in a room 14 m long, 12 m broad, and 8 m
high.
Solution
Length of the longest pole = Length of the
                      diagonal of the room
            = √(142 + 122 + 82)
            = √ 404 = 20.09 m
Problem - 9

 Area of a rhombus is 850 cm2. If one of its

diagonal is 34 cm. Find the length of the other

diagonal.
Solution
850 = ½ * d1 * d2
    = ½ * 34 * d2
    = 17 d2
 d2 = 850 / 17
    = 50 cm
         Second diagonal = 50cm
Problem - 10
A grocer is storing small cereal boxes in large
cartons that measure 25 inches by 42 inches by 60
inches. If the measurement of each small cereal
box is 7 inches by 6 inches by 5 inches then what is
maximum number of small cereal boxes that can be
placed in each large carton ?
Solution

No. of Boxes = 25*42*60 / 7*6*5 = 300

300 boxes of cereal box can be placed.
Problem - 11

If the radius of a circle is diminished by 10%,
what is the change in area in percentage?
Solution
= x + y + xy/100
= -10 - 10 + 10*10/100
= -19%
      Diminished area = 19%.
Problem - 12


A circular wire of radius 42 cm is bent in the

form of a rectangle whose sides are in the ratio

of 6:5. Find the smaller side of the rectangle?
Solution
length of wire = 2 πr = (22/7*14*14)cm
                       = 264cm
Perimeter of Rectangle = 2(6x+5x) cm
                        = 22xcm
                   22x =264         x = 12 cm
           Smaller side = (5*12) cm = 60 cm
Problem - 13
A beam 9m long, 40cm wide and 20cm deep is
made up of iron which weights 50 kg per cubic
metre. Find the weight of the Beam.
Solution

Vol. of the Beam = lbh = 9*40/100*10/100
                  = 72 m3
Weight of the iron beam is given as lm3 = 50 kg

72/100 m3 = 72/100*50 = 36 kg
Problem - 14

If the length of a rectangle is reduced by 20%
and breadth is increased by 20%. What is the
percentage change in the area?
Solution
     x + y + (xy/100)%
     = - 20 + 20 – 400/100
     = -4
The area would decrease by 4%
Problem - 15
Find the number of bricks measuring 25 cm in
length, 5 cm is breadth and 10 cm in height for a
wall 40 m long, 75 cm broad and 5 metres in
height?
Solution
Vol. of the wall = 40*72/100*5 = 150 m3
Vol. of 1 bricks = 25/100*5/100*10/100
                 = 1/80 m3
Number of bricks required = 150/1/800
                           = 150*800
                           = 120000
Calendar
Calendar
Odd days:
0 = Sunday
1 = Monday
2 = Tuesday
3 = Wednesday
4 = Thursday
5 = Friday
6 = Saturday
Calendar
Month code: Ordinary year
J=0                           F=3
M=3                          A=6
M=1                          J=4
J=6                          A=2
S=5                          O=0
N=3                          D=5
      Month code for leap year after Feb. add 1.
Calendar
Ordinary year = (A + B + C + D )-2
             -----------------------take remainder
                        7


Leap year = (A + B + C + D) – 3
          ------------------------- take remainder
                        7
Problem - 1
11th January 1997 was a Sunday. What day of
the week on 7th January 2000?
Solution
11th Jan 1997 = Sunday
11th Jan 1998 = Monday
11th Jan 1999 = Tuesday
11th Jan 2000 = Wednesday
7th Jan 2000 is on Saturday
Problem - 2
What day of the week was on 5th June 1999?
Solution
A+B+C+D – 2 / 7
A = 1999/7 = 4
B = 1999/4 = 499/7 = 2
C = June = 4
D = 5/7 = 5
= 4+2+4+5 – 2/7 = 13/7 = 5 = Saturday
Problem - 3
On what dates of August 1988 did Friday fall?
Solution
A = 1988 / 7 = 0
B = 1988/4 = 497/7 = 0
C=3
D=x
0+0+3+x+3/7 = x/7 = 5(Friday)
Friday falls on = 5,12,19,26
Problem - 4
India got independence on 15 August 1947.
What was the day of the week?
Solution
A = 1947/7 = 1
B = 1947/4 = 486/7 = 3
C = 15/7 = 1
D=2
1+3+1+2 – 2 /7 = 5/7 = Friday
Problem - 5
7th January 1992 was Tuesday. Find the day of
the week on the same date after 5 years. i.e on
7th January 1997.
Solution
7th January 1992 = Tuesday
7th January 1993 = Thursday (Leap)
7th January 1994 = Friday
7th January 1995 = Saturday
7th January 1996 = Monday ( Leap)
7th January 1997 = Tuesday
Problem - 6
The first Republic day of India was celebrated on
26th January 1950. What was the day of the
week on that date?
Solution
A = 1950/7 = 4
B = 1950/4 = 487/7 = 4
C=0
D = 26/7 = 5
4+4+0+5 – 2/7 = 11/7 = 4 = Thursday
Problem - 7
Find the Number of times 29th day of the month
occurs in 400 consecutive year?
Solution
1 year = 1 (Ordinary Year)
1 year = 12 (Leap Year)
400 years = 97 leap year
97 * 12 = 1164
303*11 = 3333
= 1164+3333 = 4497 times
Problem - 8
If 2nd March 1994 was on Wednesday, 25 Jan
1994 was on,
Solution
A = 1994/7 = 6
B = 1994/4 = 498/7 = 1
C=0
D = 25/7 = 4
= 6 + 1 + 0 + 4 – 2 / 7 = 3 = Tuesday
Problem - 9
Calendar for 2000 will serve also?
Solution
= 2000 + 2001 + 2002 + 2003 + 2004
= 2 + 1 + 1 + 1 + 2 = 7 (Complete Week)
2005
Problem - 10
If Pinky‟s 1st birthday fell in Jan 1988 on one of
the Monday‟s, the day on which are was born is,
Solution
Jan = 1988 = Monday
Jan = 1987 = Sunday
Problem - 11
Akshaya celebrated her 60th birthday on Feb 24,
2000. What was the day?
Solution
A = 2000 /7 = 7
B = 2000/4 = 500/7 = 3
C=3
D = 24/7 = 0
= 7+3+3+0-3/7 = 10/7 = 3 = Wednesday
Problem - 12
On what dates of April 2008 did Sunday Fall?
Solution
Calculate for 1st April 2008
A = 2008/7 = 6
B = 2008/4 = 502/7 = 5
C = 1/7 = 1
D=0
= 6+5+1+0 – 3/ 7 = 2 = Tuesday
1st April on Tuesday, then 1st Sunday fall on 6.
Sunday falls on 6, 13, 20, 27.
Problem - 13
Today is Friday. After 62 days it will be,
Solution
62 / 7 = 6 days after Friday then it will be
  Tuesday
Problem - 14
What will be the day of the week on 1st Jan
2010?
Solution
A=1
B=5
C=0
D=1
= 1+5+0+1 – 2/ 7 = 5/7 = 5 = Friday
Problem - 15

What is the day of the week on 30/09/2007?
Calendar
Solution:
A = 2007 / 7 = 5
B = 2007 / 4 = 501 / 7 = 4
C = 30 / 7 = 2
D=5
                   ( A + B + C + D )-2
          =      -----------------------
                           7
         =        ( 5 + 4 + 2 + 5) -2
               ----------------------- = 14/7 = 0 = Sunday
                           7
Clock
Clocks
Clock:

Angle between hour hand and minute hand
 = (11m/2) – 30h

Angle between minute hand and hour hand
 =30h – (11m/2)
Problem - 1

What is the angle between the minute hand and
hour hand when the time is 2.15?
Solution
= 11 m/2 – 30(h)
   = 11 15/2 – 30(2)
   = 11(7.5) – 60
   = 82.5 – 60 = 22 1/2
Problem - 2

At what time between 5 and 6 o‟clock the hands
of a clock coincide?
Solution
Coinciding Angle = 0
Min. hand to hour hand = 25 min apart
60/55*25 = 12/11 * 25 = 300/11
           = 27 3/11min past 5
Problem - 3

At what time between 12 and 1 o‟clock both the
hands will be at right angles?
Solution
Right angle = 90 degrees
  = 30(h) – 11 m/2
90 = 30(12) – 11 m/2
180 = 360 – 11m
11m = 360 – 180
M = 180/11
16 4/11 past 12
Problem - 4

Find at what time between 7 and 8 o‟clock will
the hands of a clock be in the same straight line
but not together?
Solution

Minute hand to hour hand = 35 min apart
Straight line not together = 30 min apart
Difference = 35 – 30 = 5 min
= 60/55*5 = 12/11*5 = 60/11
= 55 5 / 11 past 7
Problem - 5

At what time between 5 and 6 are the hands of
the clock 7 minutes apart?
Solution
7 min space behind the hour hand:
25 min – 7 min = 18 min
60/55 *18 = 216/11 = 19 7/11 min past 5
7 Min space ahead the hour hand
25 min + 7 min = 32 min
60/55*32 = 12/11*32 = 384/11
= 34 10/11 min past 5
Problem - 6

A clock strikes 4 and takes 9 seconds. In order to
strike 12 at the same rate what will be the time
taken?
Solution
     Strike           Sec
      3 (interval)    9
     11               x
3x = 11*9
X = 11*9/3 = 33 Sec

More Related Content

What's hot

The complete book_of_number_system1
The complete book_of_number_system1The complete book_of_number_system1
The complete book_of_number_system1abhi_abhi22
 
Questions on average, percentage, profit
Questions on average, percentage,  profitQuestions on average, percentage,  profit
Questions on average, percentage, profitDr. Trilok Kumar Jain
 
Questions Of Quantitative Aptitude Tests For Competitive Examinations
Questions Of Quantitative Aptitude Tests For Competitive ExaminationsQuestions Of Quantitative Aptitude Tests For Competitive Examinations
Questions Of Quantitative Aptitude Tests For Competitive ExaminationsDr. Trilok Kumar Jain
 
Number systems level 2 special equations 4
Number systems level  2 special equations 4Number systems level  2 special equations 4
Number systems level 2 special equations 4Veeraragavan Subramaniam
 
500 most asked apti ques in tcs, wipro, infos(105pgs)
500 most asked apti ques in tcs, wipro, infos(105pgs)500 most asked apti ques in tcs, wipro, infos(105pgs)
500 most asked apti ques in tcs, wipro, infos(105pgs)PRIYANKKATIYAR2
 
Tcs 2011with solutions
Tcs 2011with solutionsTcs 2011with solutions
Tcs 2011with solutionsAshu0711
 
Govt bank written(19-20)-converted [www.onlinebcs.com]
Govt bank written(19-20)-converted [www.onlinebcs.com]Govt bank written(19-20)-converted [www.onlinebcs.com]
Govt bank written(19-20)-converted [www.onlinebcs.com]Itmona
 
Faculty based bank written solution
Faculty based bank written solutionFaculty based bank written solution
Faculty based bank written solutionItmona
 

What's hot (16)

The complete book_of_number_system1
The complete book_of_number_system1The complete book_of_number_system1
The complete book_of_number_system1
 
Questions on average, percentage, profit
Questions on average, percentage,  profitQuestions on average, percentage,  profit
Questions on average, percentage, profit
 
Numbers standard level
Numbers standard levelNumbers standard level
Numbers standard level
 
Management Aptitude Test 18 Nov
Management Aptitude Test 18 NovManagement Aptitude Test 18 Nov
Management Aptitude Test 18 Nov
 
Management Aptitude Test 18 Nov Ii
Management Aptitude Test 18 Nov IiManagement Aptitude Test 18 Nov Ii
Management Aptitude Test 18 Nov Ii
 
Cts 2017
Cts 2017Cts 2017
Cts 2017
 
Management Aptitude Test 8 Nov Ii
Management Aptitude Test 8 Nov IiManagement Aptitude Test 8 Nov Ii
Management Aptitude Test 8 Nov Ii
 
Questions Of Quantitative Aptitude Tests For Competitive Examinations
Questions Of Quantitative Aptitude Tests For Competitive ExaminationsQuestions Of Quantitative Aptitude Tests For Competitive Examinations
Questions Of Quantitative Aptitude Tests For Competitive Examinations
 
Amcat 3-DOWNLOAD ENABLED
Amcat 3-DOWNLOAD ENABLEDAmcat 3-DOWNLOAD ENABLED
Amcat 3-DOWNLOAD ENABLED
 
Number systems level 2 special equations 4
Number systems level  2 special equations 4Number systems level  2 special equations 4
Number systems level 2 special equations 4
 
Amcat 2
Amcat 2Amcat 2
Amcat 2
 
500 most asked apti ques in tcs, wipro, infos(105pgs)
500 most asked apti ques in tcs, wipro, infos(105pgs)500 most asked apti ques in tcs, wipro, infos(105pgs)
500 most asked apti ques in tcs, wipro, infos(105pgs)
 
Tcs 2011with solutions
Tcs 2011with solutionsTcs 2011with solutions
Tcs 2011with solutions
 
Govt bank written(19-20)-converted [www.onlinebcs.com]
Govt bank written(19-20)-converted [www.onlinebcs.com]Govt bank written(19-20)-converted [www.onlinebcs.com]
Govt bank written(19-20)-converted [www.onlinebcs.com]
 
Faculty based bank written solution
Faculty based bank written solutionFaculty based bank written solution
Faculty based bank written solution
 
Cat,mat,gmat, questions
Cat,mat,gmat, questions Cat,mat,gmat, questions
Cat,mat,gmat, questions
 

Viewers also liked

Shortcuts in Mathematics for CAT, CET, GRE, GMAT or any similar competitive ...
Shortcuts in  Mathematics for CAT, CET, GRE, GMAT or any similar competitive ...Shortcuts in  Mathematics for CAT, CET, GRE, GMAT or any similar competitive ...
Shortcuts in Mathematics for CAT, CET, GRE, GMAT or any similar competitive ...paijayant
 
Aptitude questions all topics (1)
Aptitude questions all topics (1)Aptitude questions all topics (1)
Aptitude questions all topics (1)SnehaNallabheem
 
quantitative aptitude/ objective arithmetic
quantitative aptitude/ objective arithmeticquantitative aptitude/ objective arithmetic
quantitative aptitude/ objective arithmeticteddy bear
 
Averages
AveragesAverages
Averages303303
 
50 useful quant formula
50 useful quant formula50 useful quant formula
50 useful quant formulaChintan Leo
 
Quantitative Aptitude Test (QAT)-Tips & Tricks
Quantitative Aptitude Test (QAT)-Tips & TricksQuantitative Aptitude Test (QAT)-Tips & Tricks
Quantitative Aptitude Test (QAT)-Tips & Trickscocubes_learningcalendar
 
Problem solving
Problem solvingProblem solving
Problem solvingDiksi5
 
Maths formula by viveksingh698@gmail.com
Maths formula by viveksingh698@gmail.comMaths formula by viveksingh698@gmail.com
Maths formula by viveksingh698@gmail.comvivek698
 
Natural numbers
Natural numbersNatural numbers
Natural numbersmstf mstf
 
Mathematics for Primary School Teachers. Unit 2: Numeration
 Mathematics for Primary School Teachers. Unit 2: Numeration Mathematics for Primary School Teachers. Unit 2: Numeration
Mathematics for Primary School Teachers. Unit 2: NumerationSaide OER Africa
 
New media and current affairs
New media and current affairsNew media and current affairs
New media and current affairsSanjana Hattotuwa
 
Current affairs (2011)
Current affairs (2011)Current affairs (2011)
Current affairs (2011)Wai-Kwok Wong
 
High School Placement Test: Quantitative Skills
High School Placement Test: Quantitative SkillsHigh School Placement Test: Quantitative Skills
High School Placement Test: Quantitative SkillsGregory Tewksbury
 
Probability - Question Bank for Class/Grade 10 maths.
Probability - Question Bank for Class/Grade 10 maths.Probability - Question Bank for Class/Grade 10 maths.
Probability - Question Bank for Class/Grade 10 maths.Let's Tute
 

Viewers also liked (20)

Shortcuts in Mathematics for CAT, CET, GRE, GMAT or any similar competitive ...
Shortcuts in  Mathematics for CAT, CET, GRE, GMAT or any similar competitive ...Shortcuts in  Mathematics for CAT, CET, GRE, GMAT or any similar competitive ...
Shortcuts in Mathematics for CAT, CET, GRE, GMAT or any similar competitive ...
 
Aptitude questions all topics (1)
Aptitude questions all topics (1)Aptitude questions all topics (1)
Aptitude questions all topics (1)
 
quantitative aptitude/ objective arithmetic
quantitative aptitude/ objective arithmeticquantitative aptitude/ objective arithmetic
quantitative aptitude/ objective arithmetic
 
Averages
AveragesAverages
Averages
 
Apt maths shortcuts
Apt maths shortcutsApt maths shortcuts
Apt maths shortcuts
 
50 useful quant formula
50 useful quant formula50 useful quant formula
50 useful quant formula
 
Quantitative Aptitude Test (QAT)-Tips & Tricks
Quantitative Aptitude Test (QAT)-Tips & TricksQuantitative Aptitude Test (QAT)-Tips & Tricks
Quantitative Aptitude Test (QAT)-Tips & Tricks
 
Math tricks
Math tricks Math tricks
Math tricks
 
Nift ug design sample paper gat3
Nift ug design sample paper gat3Nift ug design sample paper gat3
Nift ug design sample paper gat3
 
Certificate MOOC IFP_Prasanth Kumar
Certificate MOOC IFP_Prasanth KumarCertificate MOOC IFP_Prasanth Kumar
Certificate MOOC IFP_Prasanth Kumar
 
Problem solving
Problem solvingProblem solving
Problem solving
 
Maths formula by viveksingh698@gmail.com
Maths formula by viveksingh698@gmail.comMaths formula by viveksingh698@gmail.com
Maths formula by viveksingh698@gmail.com
 
German Tutorial II
German Tutorial IIGerman Tutorial II
German Tutorial II
 
Number Theory
Number TheoryNumber Theory
Number Theory
 
Natural numbers
Natural numbersNatural numbers
Natural numbers
 
Mathematics for Primary School Teachers. Unit 2: Numeration
 Mathematics for Primary School Teachers. Unit 2: Numeration Mathematics for Primary School Teachers. Unit 2: Numeration
Mathematics for Primary School Teachers. Unit 2: Numeration
 
New media and current affairs
New media and current affairsNew media and current affairs
New media and current affairs
 
Current affairs (2011)
Current affairs (2011)Current affairs (2011)
Current affairs (2011)
 
High School Placement Test: Quantitative Skills
High School Placement Test: Quantitative SkillsHigh School Placement Test: Quantitative Skills
High School Placement Test: Quantitative Skills
 
Probability - Question Bank for Class/Grade 10 maths.
Probability - Question Bank for Class/Grade 10 maths.Probability - Question Bank for Class/Grade 10 maths.
Probability - Question Bank for Class/Grade 10 maths.
 

Similar to Numerical reasoning I

GRADE 5 SESSION 5.pptx
GRADE 5 SESSION 5.pptxGRADE 5 SESSION 5.pptx
GRADE 5 SESSION 5.pptxLuisSalenga1
 
Multiple choice qestions
Multiple choice qestionsMultiple choice qestions
Multiple choice qestionsRejan Chitrakar
 
GRADE 6 SESSION 2.pptx
GRADE 6 SESSION 2.pptxGRADE 6 SESSION 2.pptx
GRADE 6 SESSION 2.pptxLuisSalenga1
 
Quantitative Aptitude & Mathematics 11 October
Quantitative Aptitude & Mathematics 11 OctoberQuantitative Aptitude & Mathematics 11 October
Quantitative Aptitude & Mathematics 11 OctoberDr. Trilok Kumar Jain
 
Statistics For Management 16 October
Statistics For Management 16  OctoberStatistics For Management 16  October
Statistics For Management 16 OctoberDr. Trilok Kumar Jain
 
Statistics, Index Numbers, And Analysis For Business 17 October
Statistics, Index Numbers, And Analysis For Business 17 OctoberStatistics, Index Numbers, And Analysis For Business 17 October
Statistics, Index Numbers, And Analysis For Business 17 OctoberDr. Trilok Kumar Jain
 
Practice questions and tips in business mathematics
Practice questions and tips in business mathematicsPractice questions and tips in business mathematics
Practice questions and tips in business mathematicsDr. Trilok Kumar Jain
 
Practice questions and tips in business mathematics
Practice questions and tips in business mathematicsPractice questions and tips in business mathematics
Practice questions and tips in business mathematicsDr. Trilok Kumar Jain
 
Rationalization-CE-2-Mathematics-Pre-test-1.pptx
Rationalization-CE-2-Mathematics-Pre-test-1.pptxRationalization-CE-2-Mathematics-Pre-test-1.pptx
Rationalization-CE-2-Mathematics-Pre-test-1.pptxdudaynorombaba
 
405 math solved by md.mutaher hussain [www.onlinebcs.com]
405 math solved by md.mutaher hussain [www.onlinebcs.com]405 math solved by md.mutaher hussain [www.onlinebcs.com]
405 math solved by md.mutaher hussain [www.onlinebcs.com]Itmona
 
Quantitative Aptitude Shortcut tricks
Quantitative Aptitude Shortcut tricksQuantitative Aptitude Shortcut tricks
Quantitative Aptitude Shortcut tricksBankExamsToday.com
 
H O W T O A N A L Y S E A B U S I N E S S C A S E
H O W  T O  A N A L Y S E  A  B U S I N E S S  C A S EH O W  T O  A N A L Y S E  A  B U S I N E S S  C A S E
H O W T O A N A L Y S E A B U S I N E S S C A S EDr. Trilok Kumar Jain
 
Business Mathematics And Quantitative Aptitude
Business Mathematics And Quantitative AptitudeBusiness Mathematics And Quantitative Aptitude
Business Mathematics And Quantitative AptitudeDr. Trilok Kumar Jain
 
B A S I C S T A T I S T I C S F O R N O N C O M M E R C E S T U D E N T S
B A S I C  S T A T I S T I C S  F O R  N O N  C O M M E R C E  S T U D E N T SB A S I C  S T A T I S T I C S  F O R  N O N  C O M M E R C E  S T U D E N T S
B A S I C S T A T I S T I C S F O R N O N C O M M E R C E S T U D E N T SDr. Trilok Kumar Jain
 

Similar to Numerical reasoning I (20)

GRADE 5 SESSION 5.pptx
GRADE 5 SESSION 5.pptxGRADE 5 SESSION 5.pptx
GRADE 5 SESSION 5.pptx
 
Multiple choice qestions
Multiple choice qestionsMultiple choice qestions
Multiple choice qestions
 
GRADE 6 SESSION 2.pptx
GRADE 6 SESSION 2.pptxGRADE 6 SESSION 2.pptx
GRADE 6 SESSION 2.pptx
 
Quantitative Aptitude & Mathematics 11 October
Quantitative Aptitude & Mathematics 11 OctoberQuantitative Aptitude & Mathematics 11 October
Quantitative Aptitude & Mathematics 11 October
 
Quantitative Aptitude 7 October Ii
Quantitative Aptitude 7 October IiQuantitative Aptitude 7 October Ii
Quantitative Aptitude 7 October Ii
 
Statistics For Management 16 October
Statistics For Management 16  OctoberStatistics For Management 16  October
Statistics For Management 16 October
 
Statistics, Index Numbers, And Analysis For Business 17 October
Statistics, Index Numbers, And Analysis For Business 17 OctoberStatistics, Index Numbers, And Analysis For Business 17 October
Statistics, Index Numbers, And Analysis For Business 17 October
 
Practice questions and tips in business mathematics
Practice questions and tips in business mathematicsPractice questions and tips in business mathematics
Practice questions and tips in business mathematics
 
Practice questions and tips in business mathematics
Practice questions and tips in business mathematicsPractice questions and tips in business mathematics
Practice questions and tips in business mathematics
 
Rationalization-CE-2-Mathematics-Pre-test-1.pptx
Rationalization-CE-2-Mathematics-Pre-test-1.pptxRationalization-CE-2-Mathematics-Pre-test-1.pptx
Rationalization-CE-2-Mathematics-Pre-test-1.pptx
 
Maths kiassu
Maths kiassuMaths kiassu
Maths kiassu
 
Questions on ratio and proportion
Questions on ratio and proportion Questions on ratio and proportion
Questions on ratio and proportion
 
Questions on ratio and proportion
Questions on ratio and proportion Questions on ratio and proportion
Questions on ratio and proportion
 
405 math solved by md.mutaher hussain [www.onlinebcs.com]
405 math solved by md.mutaher hussain [www.onlinebcs.com]405 math solved by md.mutaher hussain [www.onlinebcs.com]
405 math solved by md.mutaher hussain [www.onlinebcs.com]
 
Numeric entry
Numeric entryNumeric entry
Numeric entry
 
Quantitative Aptitude Shortcut tricks
Quantitative Aptitude Shortcut tricksQuantitative Aptitude Shortcut tricks
Quantitative Aptitude Shortcut tricks
 
H O W T O A N A L Y S E A B U S I N E S S C A S E
H O W  T O  A N A L Y S E  A  B U S I N E S S  C A S EH O W  T O  A N A L Y S E  A  B U S I N E S S  C A S E
H O W T O A N A L Y S E A B U S I N E S S C A S E
 
Business Mathematics And Quantitative Aptitude
Business Mathematics And Quantitative AptitudeBusiness Mathematics And Quantitative Aptitude
Business Mathematics And Quantitative Aptitude
 
B A S I C S T A T I S T I C S F O R N O N C O M M E R C E S T U D E N T S
B A S I C  S T A T I S T I C S  F O R  N O N  C O M M E R C E  S T U D E N T SB A S I C  S T A T I S T I C S  F O R  N O N  C O M M E R C E  S T U D E N T S
B A S I C S T A T I S T I C S F O R N O N C O M M E R C E S T U D E N T S
 
aptitude presentation.pptx
aptitude presentation.pptxaptitude presentation.pptx
aptitude presentation.pptx
 

More from Prasanth Kumar RAGUPATHY

Advanced Machining Processes - Unconventional Machining Processes
Advanced Machining Processes - Unconventional Machining ProcessesAdvanced Machining Processes - Unconventional Machining Processes
Advanced Machining Processes - Unconventional Machining ProcessesPrasanth Kumar RAGUPATHY
 
NOTIFICATION ON SECOND RESCHEDULING OF EXAM DATES FOR AFFILIATED COLLEGES MAY...
NOTIFICATION ON SECOND RESCHEDULING OF EXAM DATES FOR AFFILIATED COLLEGES MAY...NOTIFICATION ON SECOND RESCHEDULING OF EXAM DATES FOR AFFILIATED COLLEGES MAY...
NOTIFICATION ON SECOND RESCHEDULING OF EXAM DATES FOR AFFILIATED COLLEGES MAY...Prasanth Kumar RAGUPATHY
 
Computer aided manufacturing robotic systems
Computer aided manufacturing robotic systemsComputer aided manufacturing robotic systems
Computer aided manufacturing robotic systemsPrasanth Kumar RAGUPATHY
 

More from Prasanth Kumar RAGUPATHY (20)

Technical Dictionary English-German
Technical Dictionary English-GermanTechnical Dictionary English-German
Technical Dictionary English-German
 
German Tutorial I
German Tutorial IGerman Tutorial I
German Tutorial I
 
Advanced Machining Processes - Unconventional Machining Processes
Advanced Machining Processes - Unconventional Machining ProcessesAdvanced Machining Processes - Unconventional Machining Processes
Advanced Machining Processes - Unconventional Machining Processes
 
Advanced Manufacturing Choices
Advanced Manufacturing ChoicesAdvanced Manufacturing Choices
Advanced Manufacturing Choices
 
5 S
5 S5 S
5 S
 
Powder Metal Applications
Powder Metal ApplicationsPowder Metal Applications
Powder Metal Applications
 
Power control for hybrid motorcycle
Power control for hybrid motorcyclePower control for hybrid motorcycle
Power control for hybrid motorcycle
 
HYBRID MOTORCYCLE
HYBRID MOTORCYCLEHYBRID MOTORCYCLE
HYBRID MOTORCYCLE
 
NOTIFICATION ON SECOND RESCHEDULING OF EXAM DATES FOR AFFILIATED COLLEGES MAY...
NOTIFICATION ON SECOND RESCHEDULING OF EXAM DATES FOR AFFILIATED COLLEGES MAY...NOTIFICATION ON SECOND RESCHEDULING OF EXAM DATES FOR AFFILIATED COLLEGES MAY...
NOTIFICATION ON SECOND RESCHEDULING OF EXAM DATES FOR AFFILIATED COLLEGES MAY...
 
GRE High Frequency Words
GRE High Frequency WordsGRE High Frequency Words
GRE High Frequency Words
 
Design of transmission systems qb
Design of transmission systems qbDesign of transmission systems qb
Design of transmission systems qb
 
Me2351 gas dynamics and jet propulsion-qb
Me2351 gas dynamics and jet propulsion-qbMe2351 gas dynamics and jet propulsion-qb
Me2351 gas dynamics and jet propulsion-qb
 
Finite element analysis qb
Finite element analysis qbFinite element analysis qb
Finite element analysis qb
 
Me2354 automobile engineering-qb
Me2354 automobile engineering-qbMe2354 automobile engineering-qb
Me2354 automobile engineering-qb
 
Mg2351. principles of management 2mark
Mg2351. principles of management  2markMg2351. principles of management  2mark
Mg2351. principles of management 2mark
 
MG2351-PRINCIPLES OF MANAGEMENT
MG2351-PRINCIPLES OF MANAGEMENTMG2351-PRINCIPLES OF MANAGEMENT
MG2351-PRINCIPLES OF MANAGEMENT
 
Computer aided manufacturing robotic systems
Computer aided manufacturing robotic systemsComputer aided manufacturing robotic systems
Computer aided manufacturing robotic systems
 
Industrial robotics
Industrial roboticsIndustrial robotics
Industrial robotics
 
Anna University UG Project Report Format
Anna University UG Project Report FormatAnna University UG Project Report Format
Anna University UG Project Report Format
 
Bhel 3
Bhel 3Bhel 3
Bhel 3
 

Recently uploaded

Blowin' in the Wind of Caste_ Bob Dylan's Song as a Catalyst for Social Justi...
Blowin' in the Wind of Caste_ Bob Dylan's Song as a Catalyst for Social Justi...Blowin' in the Wind of Caste_ Bob Dylan's Song as a Catalyst for Social Justi...
Blowin' in the Wind of Caste_ Bob Dylan's Song as a Catalyst for Social Justi...DhatriParmar
 
Narcotic and Non Narcotic Analgesic..pdf
Narcotic and Non Narcotic Analgesic..pdfNarcotic and Non Narcotic Analgesic..pdf
Narcotic and Non Narcotic Analgesic..pdfPrerana Jadhav
 
Congestive Cardiac Failure..presentation
Congestive Cardiac Failure..presentationCongestive Cardiac Failure..presentation
Congestive Cardiac Failure..presentationdeepaannamalai16
 
Daily Lesson Plan in Mathematics Quarter 4
Daily Lesson Plan in Mathematics Quarter 4Daily Lesson Plan in Mathematics Quarter 4
Daily Lesson Plan in Mathematics Quarter 4JOYLYNSAMANIEGO
 
Multi Domain Alias In the Odoo 17 ERP Module
Multi Domain Alias In the Odoo 17 ERP ModuleMulti Domain Alias In the Odoo 17 ERP Module
Multi Domain Alias In the Odoo 17 ERP ModuleCeline George
 
Textual Evidence in Reading and Writing of SHS
Textual Evidence in Reading and Writing of SHSTextual Evidence in Reading and Writing of SHS
Textual Evidence in Reading and Writing of SHSMae Pangan
 
Decoding the Tweet _ Practical Criticism in the Age of Hashtag.pptx
Decoding the Tweet _ Practical Criticism in the Age of Hashtag.pptxDecoding the Tweet _ Practical Criticism in the Age of Hashtag.pptx
Decoding the Tweet _ Practical Criticism in the Age of Hashtag.pptxDhatriParmar
 
31 ĐỀ THI THỬ VÀO LỚP 10 - TIẾNG ANH - FORM MỚI 2025 - 40 CÂU HỎI - BÙI VĂN V...
31 ĐỀ THI THỬ VÀO LỚP 10 - TIẾNG ANH - FORM MỚI 2025 - 40 CÂU HỎI - BÙI VĂN V...31 ĐỀ THI THỬ VÀO LỚP 10 - TIẾNG ANH - FORM MỚI 2025 - 40 CÂU HỎI - BÙI VĂN V...
31 ĐỀ THI THỬ VÀO LỚP 10 - TIẾNG ANH - FORM MỚI 2025 - 40 CÂU HỎI - BÙI VĂN V...Nguyen Thanh Tu Collection
 
ESP 4-EDITED.pdfmmcncncncmcmmnmnmncnmncmnnjvnnv
ESP 4-EDITED.pdfmmcncncncmcmmnmnmncnmncmnnjvnnvESP 4-EDITED.pdfmmcncncncmcmmnmnmncnmncmnnjvnnv
ESP 4-EDITED.pdfmmcncncncmcmmnmnmncnmncmnnjvnnvRicaMaeCastro1
 
MS4 level being good citizen -imperative- (1) (1).pdf
MS4 level   being good citizen -imperative- (1) (1).pdfMS4 level   being good citizen -imperative- (1) (1).pdf
MS4 level being good citizen -imperative- (1) (1).pdfMr Bounab Samir
 
4.16.24 21st Century Movements for Black Lives.pptx
4.16.24 21st Century Movements for Black Lives.pptx4.16.24 21st Century Movements for Black Lives.pptx
4.16.24 21st Century Movements for Black Lives.pptxmary850239
 
Grade 9 Quarter 4 Dll Grade 9 Quarter 4 DLL.pdf
Grade 9 Quarter 4 Dll Grade 9 Quarter 4 DLL.pdfGrade 9 Quarter 4 Dll Grade 9 Quarter 4 DLL.pdf
Grade 9 Quarter 4 Dll Grade 9 Quarter 4 DLL.pdfJemuel Francisco
 
Measures of Position DECILES for ungrouped data
Measures of Position DECILES for ungrouped dataMeasures of Position DECILES for ungrouped data
Measures of Position DECILES for ungrouped dataBabyAnnMotar
 
Team Lead Succeed – Helping you and your team achieve high-performance teamwo...
Team Lead Succeed – Helping you and your team achieve high-performance teamwo...Team Lead Succeed – Helping you and your team achieve high-performance teamwo...
Team Lead Succeed – Helping you and your team achieve high-performance teamwo...Association for Project Management
 
Student Profile Sample - We help schools to connect the data they have, with ...
Student Profile Sample - We help schools to connect the data they have, with ...Student Profile Sample - We help schools to connect the data they have, with ...
Student Profile Sample - We help schools to connect the data they have, with ...Seán Kennedy
 
Visit to a blind student's school🧑‍🦯🧑‍🦯(community medicine)
Visit to a blind student's school🧑‍🦯🧑‍🦯(community medicine)Visit to a blind student's school🧑‍🦯🧑‍🦯(community medicine)
Visit to a blind student's school🧑‍🦯🧑‍🦯(community medicine)lakshayb543
 
week 1 cookery 8 fourth - quarter .pptx
week 1 cookery 8  fourth  -  quarter .pptxweek 1 cookery 8  fourth  -  quarter .pptx
week 1 cookery 8 fourth - quarter .pptxJonalynLegaspi2
 

Recently uploaded (20)

Mattingly "AI & Prompt Design: Large Language Models"
Mattingly "AI & Prompt Design: Large Language Models"Mattingly "AI & Prompt Design: Large Language Models"
Mattingly "AI & Prompt Design: Large Language Models"
 
Blowin' in the Wind of Caste_ Bob Dylan's Song as a Catalyst for Social Justi...
Blowin' in the Wind of Caste_ Bob Dylan's Song as a Catalyst for Social Justi...Blowin' in the Wind of Caste_ Bob Dylan's Song as a Catalyst for Social Justi...
Blowin' in the Wind of Caste_ Bob Dylan's Song as a Catalyst for Social Justi...
 
Narcotic and Non Narcotic Analgesic..pdf
Narcotic and Non Narcotic Analgesic..pdfNarcotic and Non Narcotic Analgesic..pdf
Narcotic and Non Narcotic Analgesic..pdf
 
Congestive Cardiac Failure..presentation
Congestive Cardiac Failure..presentationCongestive Cardiac Failure..presentation
Congestive Cardiac Failure..presentation
 
Daily Lesson Plan in Mathematics Quarter 4
Daily Lesson Plan in Mathematics Quarter 4Daily Lesson Plan in Mathematics Quarter 4
Daily Lesson Plan in Mathematics Quarter 4
 
Multi Domain Alias In the Odoo 17 ERP Module
Multi Domain Alias In the Odoo 17 ERP ModuleMulti Domain Alias In the Odoo 17 ERP Module
Multi Domain Alias In the Odoo 17 ERP Module
 
Textual Evidence in Reading and Writing of SHS
Textual Evidence in Reading and Writing of SHSTextual Evidence in Reading and Writing of SHS
Textual Evidence in Reading and Writing of SHS
 
Decoding the Tweet _ Practical Criticism in the Age of Hashtag.pptx
Decoding the Tweet _ Practical Criticism in the Age of Hashtag.pptxDecoding the Tweet _ Practical Criticism in the Age of Hashtag.pptx
Decoding the Tweet _ Practical Criticism in the Age of Hashtag.pptx
 
prashanth updated resume 2024 for Teaching Profession
prashanth updated resume 2024 for Teaching Professionprashanth updated resume 2024 for Teaching Profession
prashanth updated resume 2024 for Teaching Profession
 
31 ĐỀ THI THỬ VÀO LỚP 10 - TIẾNG ANH - FORM MỚI 2025 - 40 CÂU HỎI - BÙI VĂN V...
31 ĐỀ THI THỬ VÀO LỚP 10 - TIẾNG ANH - FORM MỚI 2025 - 40 CÂU HỎI - BÙI VĂN V...31 ĐỀ THI THỬ VÀO LỚP 10 - TIẾNG ANH - FORM MỚI 2025 - 40 CÂU HỎI - BÙI VĂN V...
31 ĐỀ THI THỬ VÀO LỚP 10 - TIẾNG ANH - FORM MỚI 2025 - 40 CÂU HỎI - BÙI VĂN V...
 
ESP 4-EDITED.pdfmmcncncncmcmmnmnmncnmncmnnjvnnv
ESP 4-EDITED.pdfmmcncncncmcmmnmnmncnmncmnnjvnnvESP 4-EDITED.pdfmmcncncncmcmmnmnmncnmncmnnjvnnv
ESP 4-EDITED.pdfmmcncncncmcmmnmnmncnmncmnnjvnnv
 
MS4 level being good citizen -imperative- (1) (1).pdf
MS4 level   being good citizen -imperative- (1) (1).pdfMS4 level   being good citizen -imperative- (1) (1).pdf
MS4 level being good citizen -imperative- (1) (1).pdf
 
4.16.24 21st Century Movements for Black Lives.pptx
4.16.24 21st Century Movements for Black Lives.pptx4.16.24 21st Century Movements for Black Lives.pptx
4.16.24 21st Century Movements for Black Lives.pptx
 
Grade 9 Quarter 4 Dll Grade 9 Quarter 4 DLL.pdf
Grade 9 Quarter 4 Dll Grade 9 Quarter 4 DLL.pdfGrade 9 Quarter 4 Dll Grade 9 Quarter 4 DLL.pdf
Grade 9 Quarter 4 Dll Grade 9 Quarter 4 DLL.pdf
 
Faculty Profile prashantha K EEE dept Sri Sairam college of Engineering
Faculty Profile prashantha K EEE dept Sri Sairam college of EngineeringFaculty Profile prashantha K EEE dept Sri Sairam college of Engineering
Faculty Profile prashantha K EEE dept Sri Sairam college of Engineering
 
Measures of Position DECILES for ungrouped data
Measures of Position DECILES for ungrouped dataMeasures of Position DECILES for ungrouped data
Measures of Position DECILES for ungrouped data
 
Team Lead Succeed – Helping you and your team achieve high-performance teamwo...
Team Lead Succeed – Helping you and your team achieve high-performance teamwo...Team Lead Succeed – Helping you and your team achieve high-performance teamwo...
Team Lead Succeed – Helping you and your team achieve high-performance teamwo...
 
Student Profile Sample - We help schools to connect the data they have, with ...
Student Profile Sample - We help schools to connect the data they have, with ...Student Profile Sample - We help schools to connect the data they have, with ...
Student Profile Sample - We help schools to connect the data they have, with ...
 
Visit to a blind student's school🧑‍🦯🧑‍🦯(community medicine)
Visit to a blind student's school🧑‍🦯🧑‍🦯(community medicine)Visit to a blind student's school🧑‍🦯🧑‍🦯(community medicine)
Visit to a blind student's school🧑‍🦯🧑‍🦯(community medicine)
 
week 1 cookery 8 fourth - quarter .pptx
week 1 cookery 8  fourth  -  quarter .pptxweek 1 cookery 8  fourth  -  quarter .pptx
week 1 cookery 8 fourth - quarter .pptx
 

Numerical reasoning I

  • 1. Numerical Reasoning
  • 3. Problems on Numbers Arithmetic Progression: The nth term of A.P. is given by Tn = a + (n – 1)d Sum of n terms of A.P S = n/2 *[2a+(n-1)d)] Geometrical Progression: Tn = arn – 1. Sn = a(rn – 1) / (r-1)
  • 4. Basic Formulae 1. ( a+b)2 = a2 + b2 + 2ab 2. (a-b)2 = a2 +b2 -2ab 3. ( a+b)2 - (a – b)2 = 4ab 4. (a+b)2 + (a – b)2 = 2 (a2 +b2) 5. (a2 – b2) = (a+b) (a-b) 6. (a+b+c)2 =a2 +b2 +c2 + 2(ab +bc+ca) 7. (a3 +b3) = ( a+b) (a2 –ab +b2) 8. (a3 –b3) = (a-b) (a2 +ab + b2)
  • 5. Problem - 1 A 2 digit number is 3 times the sum of its digits if 45 is added to the number. Its digits are interchanged. The sum of digits of the number is?
  • 6. Solution The number is 3 times the sum of its digits 45 is added = 4 +5 = 9 So, common numbers in 3 and 9th table. 9, 18, 27, 36, 45…. 27 + 45 = 72 2 + 7 = 9 or 4 + 5 = 9
  • 7. Problem - 2 A number when divided by 119 leaves a remainder of 19. If it is divided by 17. It will leave a remainder of?
  • 8. Solution = 19/17 = 2 remainder
  • 9. Problem - 3 A boy was asked to find the value of 3/8 of sum of money instead of multiplying the sum by 3/8 he divided it by 8/3 and then his answer by Rs.55. Find the correct answer?
  • 10. Solution 8/3 – 3/8 = 55/24 = 55/55/24 = 24
  • 11. Problem - 4 A man spends 2/5rd of his earning. 1/4th of the expenditure goes to food, 1/5th on rent, 2/5th on travel and rest on donations. If his total earning is Rs.5000, find his expenditure on donations?
  • 12. Solution 5000*2/5 = 2000 Remaining amount has given as donation 2000* (1/5 + 2/5 + ¼) Total amount = 200*17/20 = 1700 2000 – 1700 = 300
  • 13. Problem - 5 From a group of boys and girls 15 girls leave. There are then left, 2 boys for each girl. After this 45 boys leave, there are then left 5 girls for each boy, find the number of girls in the beginning?
  • 14. Solution 15 girls leave = 2 boys for each girl 45 boys leave = 5 girls fro 1 boy Let the boys be x; Girls = x/2 +15 After the boys have left, No.of boys = x – 45 and girls = 5(x-45) x/2 = 5(x-45) X = 2(5x-22) X = 10x – 450 X =50 50/2 +15 =40
  • 15. Problem - 6 An organization purchased 80 chairs fro Rs.9700. For chairs of better quality they paid Rs.140 each and for each of the lower grade chair they paid Rs.50 less. How many better quality chairs did the organization buy?
  • 16. Solution Better quality chairs = x; Lower quality = 80 –x Price of better quality = Rs.140, Lower quality = 140-50 = 90 140*x + 90(80-x) = 9700 140x + 7200 – 90x = 9700 50x = 9700 – 7200; 50x = 2500 X = 50
  • 17. Problem - 7 A labour is engaged for 30 days, on the condition that Rs.50 will be paid for everyday he works and Rs.15 will be deducted from his wages for everyday he is absent from work. At the end of 30 days he received Rs.850 in all. For how many days did he wanted?
  • 18. Solution Total wages = 30*50 = 1500 (without Absent) Wages received in 30 days = 850 (with Absent) Let the labourer work for x days Absent = 30 – x 50x – (30-x)15 = 850 50x -450 +15x = 850 65x = 1300 X = 1300/65 = 20 days
  • 19. Problem - 8 The rent is charged at Rs.50 per day for first 3 days Rs.100 per day next 5 days, and 300 per day thereafter. Registration fee is 50 at the beginning. If a person had paid Rs.1300 for his stay how many days did he stay?
  • 20. Solution 3 days = 150 + 50 = 200 5 days = 100*5 = 500 = 200 + 500 = 700 1300 – 700 = 600 2 days = 300*2 = 600 = 5 + 3 + 2 = 10 days
  • 21. Problem - 9 In a school 20% of students are under the age of 8 years. The number of girls above the age of 8 years is 2/3 of the number of boys above the age of 8 years and amount to 48. What is the total number of students in the school?
  • 22. Solution Girls above 8 yrs = 48 Boys above 8 yrs = 48 / 2/3 80% of students above 8 yrs = 48 + 72 = 120 80 120 20 x 80x = 120*20 X = 120*20/80 = 30 Total No.of students = 120+30 = 150
  • 24. Ratio and Proportion Ratio : Relationship between two variables. =a:b Proportion : Relationship between two ratios. =a:b::c:d Proportion Calculation = a*d : b*c
  • 25. Problem - 1 The ratio of number of boys to that of girls in a school is 3:2. If 20% boys and 25% of girls are scholarship holders, find the percentage of the school students who are not scholarship holders?
  • 26. Solution Let the total number of students be 100 Boys = 100*3/5 = 60 Girls = 100*2/5 = 40 S. holders = 60*20/100 = 12, non S. holders = 60 -12 = 48 Girls s. holders = 40*25/100 = 10, Non s. holders = 40 – 10 = 30 Students who do not have scholarship = 48 + 30 = 78 78/100*100 = 78%
  • 27. Problem - 2 The cost of diamond varies as the square of its weight. A diamond weighing 10 decigrams costs Rs. 32000. Find the loss incurred when it breaks into two pieces whose weights are in the ratio 2:3?
  • 28. Solution 1st piece = 10*2/5 = 4 2nd piece = 10*3/5 = 6 Cost of the diamond varies as square of its weight 42 : 62 102 = 100k 16k : 36k 100k – 52 k = 48k(loss) 100k = 32000; k = 320 48*320 = 15360
  • 29. Problem - 3 The ratio of the first and second class fares between two railway stations 4 : 1 and the ratio of the number of passengers traveling by first and second class is 1:40. If the total of Rs.1100 is collected as fare from passengers of both classes what was the amount collected from first class passengers?
  • 30. Solution Fare = 4 : 1 Passengers traveling = 1 : 40 Amount = No. pas * fare = 4*1 :10*1 = 4 : 40 = 1:10 Total amount = 1100. First class passengers‟ amount = 1*1100/11 = 100
  • 31. Problem - 4 A vessel contains a mixture of water and milk in the ratio 1:2 and another vessel contains the mixture in the ratio 3:4. Taking 1 kg each from both mixtures a new mixture is prepared. What will be the ratio of water and milk in the new mixture?
  • 32. Solution 1st vessel = water = 1/3 , milk = 2/3 2nd vessel = water = 3/7, milk = 4/7 Water = 1/3 + 3/7 = 16/21 Milk = 2/3 + 4/7 = 26/21 16 : 26 = 8:13
  • 33. Problem - 5 Ratio of the income of A, B, C last year 3 : 4 : 5. The ratio of their individual incomes of last year and this year are 4:5, 2:3 and 3:4 respectively. If the sum of their present income is Rs.78,800. Find the present individual income of A, B and C.
  • 34. Solution A‟s Present Income = 5/4*3x = 15x/4 B‟s Present Income = 3/2*4x = 12x/2 C‟s Present Income = 4/7*5x = 20x/7 15x/4 + 6x+20x/3 = 78,800 197x/12 = 78,800 X = 945600/197 X = 4,800 A‟s Present income = 15x/4 = 15*4800/4 = 18,000 B‟s Present income = 6*x = 6*4,800 = 28,800 C‟s Present income = 20x/3 = 20*4800/3 = 32,000
  • 35. Problem - 6 Of the three numbers, the ratio of the first and the second is 8:9 and that of the second and third is 3:4. If the product of the first and third numbers is 2,400, then find the second number?
  • 36. Solution a:b=8:9 b:c=3:4 b : c = 3*3 : 4*3 = 9 : 12 a : b : c = 8 : 9 : 12 Product of first and third = 8k * 12k = 2400 96k2 = 2400; k2 = 2400/96 = 25 k=5 Second number = 9 * 5 = 45
  • 37. Problem - 7 Annual income of A and B are in the ratio of 4 : 3 and their annual expenses are in the ratio 3 : 2. If each of them saves Rs.600 at the end of the year, what is the annual income of A?
  • 38. Solution Income = 4 : 3, Expenses = 3 : 2 Savings 600 each A‟s income = 4x, expenses = 3x, savings = x i.e 600 Income = 4*600 : 3*600 A : B = 2400 : 1800 A income = 2400
  • 39. Problem - 8 The property of a man was divided among his wife, son and daughter according to his will as follows. Wife‟s hare is equal to 6/7th of son‟s share and daughter share is equal of 4/7th of Son‟s. If the son and daughter together receives Rs.1,02,300. How much does his wife get?
  • 40. Solution Let the Son‟s share be x. Daughter‟s share = x*4/7 = 4x/7 Wife‟s share = x* 6/7 = 6x/7 X + 4x/7 = 1,02,300 7x + 4x = 1,02,300 X = 1,02,300 /11 = 65,100 Wife Share = 65,100 *6/7 = Rs. 55, 800
  • 41. Problem - 9 A pot containing 81 litres of pure milk of the milk 1/3 is replaced by the same amount of water. Again 1/3 of the mixture is replaced by the same amount of water. Find the ratio of milk to water in the new mixture?
  • 42. Solution Milk : Water Initial = 81 : 0 1/3 removed = 54 : 27 1/3 mixture = 36 : 45 Ratio of Milk and Water = 4 : 5
  • 43. Problem - 10 729 ml of mixture contains milk and water are in the ratio 7 : 2. How much more water is to be added to get a new mixture containing milk and water in the ratio of 7 : 3.
  • 44. Solution Water = 729 * 2/9 = 162 Ratio Water 2 162 3 x 2x = 3*162/2 = 243 243 – 161 = 81 ml water is to be added
  • 45. Problem - 11 Price of a scooter and a television set are in the ratio 3 : 2. If the scooter costs Rs.600 more than the television set, then find the price of television?
  • 46. Solution Diff. in ratio = 3 – 2 = 1 1 ratio is 600 means, the television cost is 2 ratio so, cost of television = 1200
  • 47. Problem - 12 The annual income and expenditure of man and his wife are in the ratio of 5:3 and 3:1 respectively, if they decide to save equally and find their balance is 4000. Find their income at the end of the year?
  • 48. Solution Man and Wife income = 5 : 3 = 2 (diff) Man and Wife Expenses = 3 : 1 = 2 (diff) so, both of them are saving ratio of 2 Total saving of Man and Women = 4000, individual saving 2000 So, Man income = 5000 and Women income = 3000
  • 49. Problem - 13 In a class room, ¾ of the boys are above 160 cm in height and they are in 18 number. Also out of the total strength, the boys are only 2/3 and the rest are girls. Find the total number of girls in a class?
  • 50. Solution ¾ of the boys in 18 numbers means, ¼ of the boys = 6 Total number of boys = 18+6 = 24 Ratio Number 2/3 24 1/3 x 2/3*x = 24*1/3 x = 24/2 = 12 Girls
  • 51. Problem - 14 Rs. 770 was divided among A, B and C such that A receives 2/ 9th of what B and C together receive. Find A‟s share?
  • 52. Solution A = 2/9 (B+C) B+C =9A/2 A+B+C = 770 A + 9A/2 = 770 11A = 770*2 A = 140
  • 53. Problem - 15 A sporting goods store ordered an equal number of white and yellow balls. The tennis ball company delivered 45 extra white balls making the ratio of white balls to yellow balls 1/5 : 1/6. How many white tennis balls did the store originally order for?
  • 54. Solution Let the number of yellow balls be x (x + 45) : x = 1/5 : 1/6 Solving the above equation, The number of white balls originally ordered would be = 225 balls
  • 56. Alligation and Mixture Alligation : It is the rule that enables us to find the ratio in which two or more ingredients at the given price must be mixed to produce a mixture of a desired price. (Quantity of cheaper / Quantity of costlier) (C.P. of costlier) – (Mean price) = -------------------------------------- (Mean price) – (C.P. of cheaper)
  • 57. Alligation or Mixture Cost of Cheaper Cost of costlier c d Cost of Mixture m d-m m-c (Cheaper quantity) : (Costlier quantity) = (d – m) : (m – c)
  • 58. Problem -1 Three glasses of size 3 lit, 4 lit and 5 lit contain mixture of milk and water in the ratio of 2:3, 3:7 and 4:11 respectively. The content of all the three glasses are poured into a single vessel. Find the ratio of milk and water in the resulting mixture.
  • 59. Solution 1st Vessel = Milk = 3*2/5 = 6/5 = Water = 3*3/5 = 9/5 2nd Vessel: = Milk = 4*3/10 = 12/10 = Water = 4*7/10 = 28/10 3rd Vessel: = Milk = 5*4/15 = 20/15 = Water = 5*11/15 = 55/15 Milk : Water = 6/5 +12/10 + 20/15 : 9/5 + 28/10 + 55/15 = 18/15 + 18/15 +20/15 : 27/15 + 42/15+55/15 = 56 : 124 (or) 14:31
  • 60. Problem - 2 How many kg of tea worth Rs. 25 per kg must be blended with 30 kg tea worth Rs. 30 per kg, so that by selling the blended variety at Rs.30 per kg there should be a gain of 10%?
  • 61. Solution 30*100/110 = 300/11 25 30 300/11 30/11 25/11 30 : 25 6 : 5 36 : 30kg
  • 62. Problem - 3 A man buys cows for Rs. 1350 and sells one so as to lose 6% and the other so as to gain 7.5% and on the whole he neither gains nor loses. How much does each cow cost?
  • 63. Solution 6 7.5 0 7.5 6 15 12 5 : 4 1350*5/9 = 750 1350 *4/9 = 600
  • 64. Problem - 4 There are 65 students in a class, 39 rupees are distributed among them so that each boy gets 80p and each girl gets 30p. Find the number of boys and girls in a class.
  • 65. Solution Girls Boys 30 80 60 20 30 2 : 3 65*2/5 = 26 65*3/5 = 39
  • 66. Problem - 5 A person covers a distance 100 kms in 10 hr Partly by walking at 7 km per hour and rest by running at 12 km per hour. Find the distance covered in each part.
  • 67. Solution Speed = Distance / Time = 100 / 10 = 10 7 12 10 2 : 3 Time taken in 7 km/hr = 10 * 2/5 = 4 4*7 = 28 km Time taken in 12 km/hours = 10*3/5 = 6 12*6 = 72 km
  • 68. Problem - 6 A merchant has 100 kg of salt, part of which he sells at 7% profit and the rest at 17% profit. He gains 10% on the whole. Find the quantity sold at 17% profit?
  • 69. Solution 7 17 10 (17-10) (10-7) 7 : 3 The quantity of 2nd kind = 3/10 of 100kg = 30kg
  • 70. Problem - 7 In what ratio two varieties of tea one costing Rs. 27 per kg and the other costing Rs. 32 per kg should be blended to produce a blended variety of tea worth Rs. 30 per kg. How much should be the quantity of second variety of tea, if the first variety is 60 kg?
  • 71. Solution 27 32 30 2 3 Quantity of cheaper tea = 2 Quantity of superior tea 3 Quantity of cheaper tea =2*x/5 = 60 , x=150 Quantity of superior tea = 3 * 150/5 = 90 kg
  • 72. Problem - 8 A 3-gallon mixture contains one part of S and two parts of R. In order to change it to mixture containing 25% S how much R should be added?
  • 73. Solution R : S 2 : 1 75% : 25% 3 : 1 1 gallon of R should be added.
  • 74. Problem - 9 Three types of tea A,B,C costs Rs. 95/kg, Rs. 100/kg. and Rs 70/kg respectively. How many kg of each should be blended to produce 100 kg of mixture worth Rs.90/kg given that the quantities of B and C are equal?
  • 75. Solution B+C/2 A 85 95 90 5 5 Ratio is 1:1 so A = 50 , B + C = 50 The quantity would be 50 : 25 : 25
  • 76. Problem - 10 In what proportion water must be added to spirit to gain 20% by selling it at the cost price?
  • 77. Solution Profit%=20% Let C.P =S.P= Rs.10 Then CP=100/(100+P%)SP =25/3 0 10 25/3 5/3 25/3 The ratio is 1: 5
  • 78. Problem - 11 In an examination out of 480 students 85% of the girls and 70% of the boys passed. How many boys appeared in the examination if total pass percentage was 75%
  • 79. Solution Solution: 70 85 75 10 5 Number of Boys = 480 * 10/15 Number of Boys = 320
  • 80. Problem - 12 A painter mixes blue paint with white paint so that the mixture contains 10% blue paint. In a mixture of 40 litre paint how many litre of blue paint should be added, so that the mixture contains 20% of blue paint?
  • 81. Solution Quantity of blue paint in the mixture = 10% of 40 40*10/100 = 4 40 – 4 = 36 litre Let x litre blur paint can be mixed 4+x/30 = 20/80 = 4+x = 9 x=5
  • 82. Problem - 13 From a 100 litre mixture containing water and milk equal proportion, 10 litres of mixture is replaced by 10 litres of water in succession twice. At the end, what is the ratio of milk and water?
  • 83. Solution Milk Water 10 lit(1st) 50 : 50 45 : 45 45 : 55 2nd 10 lit 40.5 : 49.5 Add Water 40.5 : 59.5 81 : 119
  • 84. Problem - 14 In a mixture of 400 gms, 80% is copper, sliver is 20%. How much copper is to be added, so that the new mixture has 84% copper?
  • 85. Solution 400*80/100 = 320 Copper 400*20/100 = 80 Sliver Percen Mixture 80 320 84 x = 320*84/80 = 336 (320+x) = (400+x) 84/100 320+x = 400+84/100 + 84x/100 16x/100 = 336 – 320; 16x/100 = 16; x = 100
  • 86. Problem - 15 A jar full of whisky contains 50% alcohol. A part of this whisky is replaced by another containing 30% alcohol and now the percentage of alcohol was found to be 35%. Find the quantity of whisky replaced?
  • 87. Solution 50 30 35 5 : 15 5 : 15 = 1 : 3 Replaced = 3/4
  • 89. Type - 1 A invest = 10000 B invest = 15000 Profit = 5000 Find their Individual Share ? A : B = 10000 : 15000 = 2 : 3 A‟s Share = 5000*2/5 = 2000 B‟s Share = 5000*3/5 = 3000 This is a first and basic step for any Partnership Problem.
  • 90. Type - 2 A invest = 5000, After 3 months B joined A, with an investment of 3000 Profit at the end of the year = 3500 Find their Share ? Any thing happen after a month, like a person joining a business, or withdraw from business or withdraw some amount means given amount is for month. Cont…
  • 91. Type - 2 A : B = 5000 : 3000 = 5*12 : 3*9 = 60 : 27 = 20 : 9 A‟s share = 3500*20/29 = 2413.7 B‟s Share = 3500*9/29 = 1086.3
  • 92. Type - 3 A invest 5000 B invest 6000 After 3 months A withdraw amount 1000, after 5 months a withdraw amount 1000 again. Profit at the end of the Year = 5000 Find their Share ? A = 5*3 + 4*5 + 3*4 = 15 +20 + 12 = 47 B = 6*12 = 72
  • 93. Type - 3 A‟s share = 5000* 47/119 = 1974.8 B‟s share = 5000*72/119 = 3025.2
  • 94. Type - 4 A invest twice as much as B, B invest 1/3rd of C. At the end of the year their Profit is 6000. Find their Share? A = 2B B = 1/3C C=x A : B : C = 2x/3 : x/3 : x A : B : C = 2x/3 : x/3 : 3x /3 A:B:C=3:2:6 A‟s Share = 6000*3/11 = 1636 B‟s Share = 6000*2/11 = 1091 C‟s Share = 6000*6/11 = 3273
  • 95. Problem - 1 A, B and C started a business in partnership by investing Rs.12000 each. After 6 months, C left and after 4 months D joined with his capital of Rs.24,000. At the end of a year, a profit of Rs.8,500 shared among all the partners. Find B‟s share?
  • 96. These are all the basic types remaining we will see when we solve problems.
  • 97. Solution A : B : C : D 12000 : 12000 : 12000 : 24000 1 : 1 : 1 : 2 1*12 : 1*12 : 1*6 : 2*2 12 : 12: 6 : 4 6 : 6 : 3 : 2 B‟s share = 6/17*8500 = 3000
  • 98. Problem - 2 A, B and C enter into partnership. A contributes one third of the capital while B contributes as much as A and C together contributed. If the profit at the end of the year amounted to Rs.840. What would be B‟s share?
  • 99. Solution A‟s share = 1/3 of the capital A‟s share = 1/3*840 = 280 B‟s share = A + C = 280 + x A + B + C = 840 280 + 280 + x + x = 840 560 + 2x = 840 2x = 840 – 560 X = 140 B‟s share = 280+140 = 420
  • 100. Problem - 3 Akilesh and Jaga enter into a partnership. Akilesh contributing Rs.8000 and Jaga contributing Rs.10000. At the end of 6 months they introduce Prakash, who contributes Rs.6000. After the lapse of 3 years, they find that he firm has made a profit of Rs.9660. Find Prakash‟s share?
  • 101. Solution Akilesh : Jaga : Prakash 8 : 10 : 6 4 : 5 : 3 4*36 : 5836 : 3*30 144 : 180 : 90 8 : 10 : 5 Prakash‟s share = 9660*5/23 = 2100
  • 102. Problem - 4 Priya and Vijay enter into partnership. Priya supplies whole of the capital amounting to Rs.45,000 with the conditions that the profit are to be equally divided and that Vijay pays Priya interest on half of the capital of 10% p.a. but receives Rs.120 per month for carrying on the concern. Find their total yearly profit. When Vijay‟s income is one half of Priya‟s income?
  • 103. Solution 45,000 *1/2 = 22,500 22,500 *10//100 = 2250 (interest p.a) Vijay receives Rs.120 per month = 120*12 = 1440 Total profit be x Ratio of Profit sharing = 1 : 1 Priya‟s income = x/2+2250 Vijay‟s income = x/2 – 2250 +1440 1 Priya = ½ Vijay Priya Income = Twice of Vijay income x/2 + 2250 = 2(x/2 – 2250 +1440) X+4500/2 = 2(x/2 – 810) X+4500/2 = x – 1620 = x +4500 = 2x – 3240 X = 7740 Total Profit of the year = 7740+1440 = 9,180
  • 104. Problem - 5 Revathy and Shiva are partners sharing profits in the ratio of 2:1. They admit Pooja into partnership giving her 1/5th share in profits which she acquires from Revathy and Shiva in the ratio of 1:2. Calculate the new profit sharing ratio?
  • 105. Solution Pooja gets her share of 1/5th of total share of Profit from Revathy and Shiva in the ratio 1 : 2 From Revathy = 1/3*1/5 = 1/15 From Shiva = 2/3*1/5 = 2/15 Total Pooja share = 1/15+2/15 = 3/15 = 1/15 Revathy share = 2/3 – 1/15 = 9/15 Shiva share = 1/3 – 2/15 = 3/15 Shares = Revathy : Shiva : Pooja = 3 : 1 : 1
  • 106. Problem - 6 A and B started a partnership business investing some amount in the ratio of 3 : 5. C joined them after six months with an amount equal to that of B. In what proportion should the profit at the end of 1 year be distributed among A, B and C?
  • 107. Solution Let the investment, 3 : 5 : 5 3*12: 5*12 : 5*6 36 : 60 : 30 6 : 10 : 5
  • 108. Problem - 7 If 4(A‟s capital) = 6(B‟s capital) = 10 (C‟s capital) then out of a profit of rs.4650. Find C‟s share?
  • 109. Solution Let the unknown value be x x/4 : x/6 : x/10 15x/60 : 10x/60 :6x/60 15 : 10 : 6 C‟s share = 6/31*4650 = Rs. 900
  • 110. Problem - 8 A, B, C subscribe Rs.50,000 fro business. A subscribes Rs.4000 more than B and B Rs.5000 more than C. Out of total profit of Rs.35,000. Find A‟s share?
  • 111. Solution C = x, B = x + 5000 A = x+5000+4000 = x + 9000 x +x+5000 +x+9000 = 50000 3x+14000 = 50000 3x = 50000 – 14000 3x = 36000, x = 12000 C : B : A 12000 : 17000 : 21000 A = 35000*21/50 = 14,700
  • 112. Problem - 9 A and B are partners in a business, A contributes ¼ of he capital for 15 months and B received 2/3 of the profit. For how long B‟s money was used?
  • 113. Solution B = 2/3 A = 1/3 A : B = 1/3 : 2/3 = 1 : 2 Investment 1/4x+15 : 3/4x*y 15x/4 : 3xy/4 15x/4 : 3xy/4 : : 1 : 2 30x/4 = 3xy/4 Y = 30x/4 * 4/3x = 10 months
  • 114. Problem - 10 A, B and C invests Rs.4,000, Rs.5,000 and Rs.6,000 respectively in a business and A gets 25% of profit for managing the business and the rest of the profit is divided by A, B and C in proportion to their investment. If in a year, A gets Rs.200 less than B and C together, what was the total profit for the year?
  • 115. Solution Total Profit = 100 25% for managing the business = 100 – 25 = 75% A : B : C 4000 : 5000 : 6000 4 : 5 : 6 4x : 5x : 6x = 25x 100*15x/75 = 20x A gets 4x + 25% of 20x = 4x + 20x *25/100 = 9x B = 5x, C = 6x (5x + 6x) – 9x = 200 11x – 9x = 200 2x = 200; x = 100 Total Profit 20x = 20*100 = 2000
  • 116. Problem - 11 A and B entered into partnership with capitals in the ratio of 4 : 5. After 3 months, A withdraw ¼ of his capital and B withdraw 1/5 of his capital. The gain at the end of 10 months was Rs.760. Find the share of B?
  • 117. Solution A : B 4 : 5 4000 : 5000 A‟s share = 4000*1/4 = 4000 – 1000 = 3000 B‟s share = 5000*1/5 = 5000 – 1000 = 4000 A : B 3*4+3*7 : 5*3 +4*7 12 + 21 : 15+28 33 : 43 60*43/76 = 430
  • 118. Problem - 12 Rs. 1290 is divided between A, B and C. So, that A‟s share is 1 ½ times B‟s and B‟s share is 1 ¾ times C. What is C‟s share?
  • 119. Solution A : B = 1 ½ : 1 = 3/2 : 1 = 3 : 2 B : C = 1 ¾ : 1 = 7/4 : 1 = 7 : 4 A :B :C =3*7(A) : 2*7(B) : 7*2(B) : 4*2(C) = 21 : 14 : 8 B = 1290*8/43 = Rs.240
  • 120. Problem - 13 A man starts a business with a capital of Rs.90000 and employs an assistant. From the yearly profit he keeps an amount equal to 4 ½ of his capital and pay 35% of the remainder of the profits. Find how much the assistant receives in a year, in which profit is Rs.30,000.
  • 121. Solution Investment = 90,000 4 ½ of investment = 9/2/100*90000 = Rs.4050 Profit = 30,000 – 4050 = 25,950 35/100*25,950 =9082.50
  • 122. Problem - 14 A and B invest in a business in the ratio 3 : 2. If 5% of the total profit goes to charity and A‟s share is Rs. 855, what is the total profit %?
  • 123. Solution Let the total profit be Rs. 100 After paying charity A‟s share = 3/5 *95 = 57 If A‟s share is Rs. 57, the total profit is 100 If A‟s share is Rs. 855, the total profit is 100 * 855/57 = Rs. 1500 The total profit = Rs. 1500
  • 124. Problem - 15 A,B,C entered into a partnership by making an investment in the ratio of 3 : 5 : 7. After a year C invested another Rs. 337600 while A withdrew Rs. 45600. The ratio of investments then changed into 24: 59 : 167. How much did A invest initially?
  • 125. Solution Solution: Let the investments of A, B, and C be 3x, 5x, 7x (3x – 45600) : 5x : (7x + 337600) = 24 : 59 : 167 (3x – 45600)/5x = 24/59 x = 47200 Initial investment of A = 47200 * 3 = Rs. 141600
  • 127. Problem - 1 The age of the Father is 4 times the age of his Son. If 5 years ago, Father‟s age was 7 times the age of his Son, what is the Father‟s present age?
  • 128. Solution F = 4S F - 5 = 7(S - 5) 4S – 5 = 7S – 35 3S = 30 S = 10 Father‟s age = 4* 10 = 40 years
  • 129. Problem - 2 The age of Mr. Gupta is four times the age of his Son. After Ten years, the age of Mr. Gupta will be only twice the age of his Son. Find the present age of Mr. Gupta‟s Son.
  • 130. Solution G = 4S G + 10 = 2 ( S + 10) 4S + 10 = 2S + 20 2S = 10 S=5 Son‟s Age = 5 years
  • 131. Problem - 3 10 years ago Anu‟s mother was 4 times older than her daughter. After 10 years, the mother will be twice as old as her daughter. Find the present age of Anu.
  • 132. Solution Ten years before: M – 10 = 4(A – 10 ) M – 10 = 4A – 40 M = 4A – 40 + 10 M = 4A – 30 Ten Years After: M + 10 = 2(A + 10) M + 10 = 2A + 20 M = 2A + 20 – 10 M = 2A + 10 4A – 30 = 2A + 10 2A = 10 + 30 2A = 40: Anu‟s Age = 20
  • 133. Problem – 4 The sum of the ages of A and B is 42 years. 3 years back, the ages of A was 5 times the age of B. Find the difference between the present ages of A and B?
  • 134. Solution A + B = 42 A = 42 – B A – 3 = 5 ( B – 3) A – 3 = 5B – 15 42 – B – 3 = 5B – 15 42 – 3 + 15 = 5B + B 54 = 6B B = 54 /6 = 9 A = 42 – B; A = 42 – 9 = 33 Difference in their ages = 33 – 9 = 24 Years
  • 135. Problem - 5 The sum of the ages of a son and father is 56 years. After 4 years, the age of the father will be 3 times that of the son. Find their respective ages?
  • 136. Solution F + S= 56 S = 56 – F F + 4 = 3 (S + 4) F + 4 = 3 (56 – F + 4) F + 4 = 168 – 3F + 12 4F = 168 + 12 – 4 4F = 176 ; F = 44 S = 56 – F ; S = 56 – 44 = 12 Father Age = 44; Son Age = 12
  • 137. Problem – 6 The ratio of the ages of father and son at present is 6:1. After 5 years, the ratio will become 7:2. Find the Present age of the son.
  • 138. Solution 6x + 5/x + 5 = 7/2 12x + 10 = 7x + 35 12x – 7x = 35 – 10 5x = 25 x = 25 / 5 x = 5 years Son age = 1* 5 = 5 years
  • 139. Problem - 7 The ages of Ram and Shyam differ by 16 years. Six years ago, Shyam‟s age was thrice as that of Ram‟s. Find their present ages?
  • 140. Solution S = R + 16 S – 6 = 3(R – 6) S – 6 = 3R – 18 R + 16 – 6 = 3R – 18 R + 10 = 3R – 18 2R = 28 ; R = 14 Shyam‟s Age = 14 + 16 = 30.
  • 141. Problem - 8 A man‟s age is 125% of what it was 10 years ago, 83 1/3% of what it will be after 10 years. What is his present age?
  • 142. Solution Let the age be x 125% of (x – 10) = 83 1/3 % of (x +10) 125/100 * x – 10 = 250/ 300 * x +10 5/4 x – 10 = 5/6 x – 10 5x / 4 – 5x / 6 = 50/6 + 50/4 5x /12 = 250/12 5x = 250 ; x = 50 years
  • 143. Problem - 9 3 years ago, the average age of a family of 5 members was 17. A baby having born, the average age of the family is the same today. What is the age of the child?
  • 144. Solution Average age of 5 members = 17 Total age of 5 members = 17*5 = 85 3 years later, the age of 5 members will be = 85 + 15 = 100 100 + x / 6 = 17 100 + x = 17*6 100 + x = 102 x = 102 – 100 = 2 years
  • 145. Problem - 10 The sum of the age of father and his son is 100 years now. 5 years ago their ages were in the ratio of 2 : 1. The ratio of the ages of father and his son after 10 years will be?
  • 146. Solution F + S = 100 5 years ago 2 : 1 5 years ago F + S = 100 – 10 = 90 90*2/3 = 60 : 30 Present age = 65 : 35 10 years ago = 75 : 45 =5:3
  • 147. Problem - 11 Six years ago, Sushil‟s age was triple the age of Snehal. Six years later, Sushil‟s age will be 5/3 of the age of Snehal. What is the present age of Snehal?
  • 148. Solution Six years ago, Snehal = x; Sushil = 3x Six years later, 3x + 6+6 = 5/3(x+6+6) 9x +36 = 5x+60 4x = 60 – 36 X=6 Present Age of Snehal = 6+6 = 12 years
  • 149. Problem - 12 Susan got married 6 years ago. Today her age is 1¼ times that at the time of her marriage. Her son is 1/6 as old as she today. What is the age of her son?
  • 150. Solution 6 years ago Susan got married. So her son‟s age will be less than 6 years. Let as consider, her son‟s age is 5 years. Susan‟s Age is 5*6 = 30 yrs, since the son is 1/6th of Susan‟s age. 6 years ago her age must have been 24 yrs 24*1 ¼ = 24*5/4 = 30 yrs As it satisfies the conditions her son‟s age is 5 years
  • 151. Problem - 13 My brother is 3 years elder to me. My father was 28 years of age when my sister was born, while my mother was 26 years of age, when I was born. If my sister was 4 years of age when my brother was born, then, what was the age of my father and mother respectively when my brother was born?
  • 152. Solution My brother was born 3 years before I was born and 4 years after my sister was born Father‟s age when brother was born = 28 + 4 = 32 years Mother‟s age when brother was born = 26 – 3 = 23 years
  • 153. Problem - 14 If 6 years are subtracted from the present age of Gagan and the reminder is divided by 18, then the present age of his grandson Aunp is obtained. If Anup is 2 years younger to Madan whose age is 5 years, then what is Gagan‟s present age?
  • 154. Solution Anup‟s age = 5 – 2 = 3 years Let Gagan‟s age be x = x – 6 / 18 = 3 x – 6 = 3*18 ; x – 6 = 54 x = 54 + 6 Gagan‟s age = 60
  • 155. Problem - 15 Ramu‟s grandfather says, “ Ram, I am now 30 years older than your father. 15 years ago, I was 2½ times as old as your father”. How old is the grandfather now?
  • 156. Solution Let the father‟s age be x. Grandfather‟s age will be 30 + x 15 years ago, X + 30 – 15 = 5/2 (x – 15) X + 15 = 5/2 (x – 15) 2x + 30 = 5x – 75 105 = 3x X = 105 / 3 = 35 Grandfather‟s age = 35 + 30 = 65
  • 158. Average Average = Sum of Quantities Number of Quantities Sum of quantities = Average*Number of Quantities. Number of quantities = Sum of Quantities Average
  • 159. Problem - 1 The average age of a class of 22 students is 21 years. The average increases by 1 when the teacher‟s age is also included. What is the age of the teacher?
  • 160. Solution Total age of the students be x x/22 = 21; x = 21*22= 462 Teacher‟s age is also included x/23 = 22; x = 22*23 = 506 Total age of 23 people – Total age of 22 people will be the age of teacher 506 – 462 = 44 years The age of teacher = 44
  • 161. Problem - 2 The average of 7 numbers is 25. The average of first three of them is 20 while the last three is 28. What must be the remaining number?
  • 162. Solution Average of 7 numbers = 25, Sum of 7 numbers = 25* 7 = 175 Avg. of first three numbers = 20, 20* 3 = 60 Avg. of last three numbers = 28, 28*3 = 84 The 4th number = 175 – (60+84) = 175 – 144 = 31
  • 163. Problem - 3 The average age of a team of 10 people remains the same as it was 3 years ago, when a young person replaces one of the member. How much younger was he than the person whose place he took?
  • 164. Solution Let Average be x 10 members‟ Average = 10x Average of 10 members (including new one) is same as it was 3 yrs ago. Now 10*3 = 30 years have increased, so a person of 30 years should have replaced to keep the average as same.
  • 165. Problem - 4 The average age of a couple was 26 years at that time of their marriage. After 11 years of marriage the average age of the family with 3 children become 19 years. What is the average age of the Children?
  • 166. Solution Average of parents ages is 26, sum= 26*2 = 52 Parents age after 11 years = 52 +22 = 74 Average age of Family = 19, Sum = 19*5 = 95 Sum of family‟s age – Sum of parents‟ age = 95 – 74 = 21 Sum of the ages of 3 children = 21, Average Age = 21/3 = 7 yrs
  • 167. Problem - 5 9 members went to a hotel for taking meals. Eight of them spent Rs. 12 each on their meals and the ninth person spent Rs. 8 more than the average expenditure of all the nine. What was the total money spent by them?
  • 168. Solution Average = x/9 Amount Spent by 8 members = 12 * 8 = 96 96 + x/9 + 8 = x 104 = x – x/9 104 = 8x/9 8x = 104 *9 = 936 x = 936/8 = 117
  • 169. Problem - 7 A batsman makes a score of 87 runs in the 17th inning and thus increases his average by 3. Find his average after 17th innings?
  • 170. Solution 17th innings avg. = x, Runs = 17x 16th innings avg. = x -3, Runs = 16 (x -3) 16 (x-3) + 87 = 17x 16x – 48 +87 = 17x X = 39
  • 171. Problem - 7 There are 24 students in a class. One of them, who was 18 yrs old left the class and his place was filled up by the newcomer. If the average of the class thereby was lowered by one month, what is the age of the newcomer?
  • 172. Solution Average reduced by 1 month, 24 * 1 = 2 years So, the newcomer‟s age is 18 -2 = 16 years
  • 173. Problem - 8 The average of marks in mathematics for 5 students was found to be 50. Later, it was discovered that in the case of one student the mark 48 was misread as 84. What is the correct average?
  • 174. Solution Difference = 84 – 48 = 36 36 /5 = 7.2 (Increased) The corrected average = 50 – 7.2 = 42.8
  • 175. Problem - 9 The average salary of all the workers in a factory is Rs. 8000. The average salary of 7 technicians is Rs. 12000 and the average salary of the rest is Rs. 6000. What is the total number of workers in the factory?
  • 176. Solution Members Avg. 7 12000 X 6000 6x = 7*12 X = 7812/6 = 14 Total no. of workers = 7 + 14 = 21
  • 177. Problem - 10 Average salary of all the 50 employees including 5 officers of the company is Rs. 850. If the average salary of the officers is 2500, find the average salary of the remaining staff of the company.
  • 178. Solution x/50 = 850; x = 42,500 5 officers‟ salary = 2500*5 = 12500 50 – 5 members = 42500 – 12500 45 members = 30000 Avg. salary of 45 members = 30000/45 = 667(App)
  • 179. Problem - 11 Find the average of 8 consecutive odd numbers 21,23,25,27,29,31,33,35
  • 180. Solution 1st number + last Number /2 = 21 + 35 /2 = 28
  • 181. Problem - 12 A train covers 50% of the journey at 30 km/hr, 25% of the journey at 25 km/hr, and the remaining at 20 km/hr. Find the average speed of the train during entire journey.
  • 182. Solution Total Journey = 100 km S = Distance / Time = 100 / 5/3 + 1/1 + 5/4 = 100 * 12 /20+12+15 = 1200/47 = 25 25/47 km/hr
  • 183. Problem - 13 The average of 10 numbers is 7. What will be the new average if each number is multiplied by 8?
  • 184. Solution If numbers are multiplied by 8, Average also to be multiplied by 8 = 7*8 = 56 {or} x/10 = 7 x = 10*7 = 70 = 70* 5 = 560 /10 = 56
  • 185. Problem - 14 The mean marks of 10 boys in a class is 70% whereas the mean marks of 15 girls is 60%. What is the mean marks of all 25 students?
  • 186. Solution Boys = x/10 = 70 = 700 Girls = x/15 = 60 = 900 10 + 15 = 700 + 900 25 = 1600 1600/25 = 64%
  • 187. Problem - 15 Of the three numbers the first is twice the second and the second is thrice the third. If the average of the three numbers is 10, what are the numbers?
  • 188. Solution A = 2x B=x C = x/3 2x + x + x/3/3 = 10 6x + 3x + x /9 = 10 6x + 3x + x = 90 10x = 90 ; x = 9. A = 18, B = 9, C = 3
  • 190. Percentage • By a certain Percent, we mean that many hundredths. • Thus, x Percent means x hundredths, written as x%
  • 191. Percentage •Finding out of Hundred. If Length is increased by X% and Breadth is decreased by Y% What is the percentage Increase or Decrease in Area of the rectangle? Formula: X+Y+ XY/100 % Decrease 20% means -20
  • 192. Problem -1 When 75% of the Number is added to 75%, the result is the same number. What is the number?
  • 193. Solution Percentage Number 75 x+75 100 x 100x + 7500 = 75x 25x = 7500 x = 300
  • 194. Problem - 2 A tank is full of milk. Half of the milk is sold and the tank is filled with water. Again half of the mixture is sold and the tank is filled with water. This operation is repeated thrice. Find the percentage of milk in the tank after the third operation?
  • 195. Solution Milk Water 100 0 50 50(1st) 25 75 (2nd) 12.5 87.5 (3rd) After 3 operation Milk 12.5%
  • 196. Problem 3 A large water-melon weighs 20kg with 96% of its weight being water. It is allowed to stand in the sun and some of the water evaporates so that now, only 95% of its weight is water. What will be its reduced weight?
  • 197. Solution 20 *96/100=19.2kg of water Let the evaporated water be x 19.2-x=95%(20-x) 19.2-x=95(20-x)/100 1920-100x=1900-95x 5x=20 ;x=4 20-4=16kg.
  • 198. Problem 4 The population of a city is 155625. For every1000 men, there are 1075 women. If 40% of men and 24% of women be literate, then what is the percentage of literate people in the city?
  • 199. Solution Ratio of men and women=1000:1075=40:43 Number of men=40*155625/83=75000 Number of women=155625-7500=80625 Number of literate men=75000*40/100=3000 Number of literate women =80625*24/100=19350 Literate people =30000+19350=49350 Percentage of literate people =49350/155625*100=2632/83=31 59/83%
  • 200. Problem 5 300 grams of sugar solution has 40% sugar in it. How much sugar should be added to make it 50% in the solution?
  • 201. Solution Grams Sugar 300 40% X 50% 50x = 40*300 x = 40*300/50 = 240 300 – 240 =60 Kg
  • 202. Problem - 6 A man lost 12½% of his money and after spending 70% of the remainder, he has Rs. 210 left. How much did the man have at first?
  • 203. Solution Let the amount be 100 Then, 100.00 – 12.50 = 87.50 70% of 87.50 = 87.50 *70/100 =61.25 The remaining amount will be Rs. 26.25 Initial Final 100 26.25 X 210 26.25x = 21000; x = 21000/26.25 = 800
  • 204. Problem - 7 During one year the population of a town increases by 10% and during next year it diminished by 10%. If at the end of the second year, the population was 89,100, what was the Population at the beginning of first year?
  • 205. Solution Let the population be 100 1st Year = 100 + 10 = 110 2nd Year = 110 * 10/100 = 110 -11 = 99 Percentage Population 99 89100 100 x 99x = 89100*100; x = 8910000/99 = 90000
  • 206. Problem - 8 When a number is first increased by 20% and then again 20% by what percent should the increase number be reduced to get back the original number?
  • 207. Solution Let the number be 100 20% increase = 100*20/100 = 20 New Value = 120 Again increase by 20% = 120*20/100 = 24 New value = 144 Increased amount = 44/144*100 = 30 5/9%
  • 208. Problem - 9 The number of students studying Arts, Commerce and Science in an institute were in the ratio 6 : 5 : 3 respectively. If the number of students in Arts, Commerce and science were increased by 10%, 30% and 15% respectively, what was the new ratio between number of students in the three streams?
  • 209. Solution A:C:S 6:5:3 6x : 5x : 3x 6x*110/100 : 5x*130/100 : 3x*115/100 6x*110 : 5x*130 : 3x*115 660 : 650 : 345 132 : 130 : 69
  • 210. Problem - 10 In measuring the sides of rectangle errors of 5% and 3% in excess are made. What is the error percent in the calculated area?
  • 211. Solution Area = xy X = 5% Excess = 100* 5/100 = 105 Y = 3% Excess = 100*3/100 = 103 103*105/100 = 10815/100 = 108.15 Error – Actual = 108.15 – 100 = 8.15% Excess
  • 212. Problem - 11 In a certain examination there were 2500 candidates. Of them 20% of them were girls and rest were boys. If 5% of boys and 40% of girls failed, what was the Percentage of candidates passed?
  • 213. Solution Girls = 2500*20/100 = 500 Boys = 2500*80/100 = 2000 Students who failed were Boys = 2000*5/100 = 100 Girls = 500*40/100 = 200 Total Failed Students = 300 Total Pass students = 2500 – 300 = 2200 Pass Percentage = 2200/2500*100 = 88%
  • 214. Problem - 12 A person saves every year 20% of his income. If his income increases every year by 10% then his saving increases by?
  • 215. Solution Every year saving, if the income is Rs. 100 = 100 *20/100 =Rs. 20 Salary increases = 110*20/100 = 22 Percentage increase (Savings) = 2/20*100 = 10%
  • 216. Problem - 13 On a test containing 150 questions carrying 1 mark each, meena answered 80% of the first answers correctly. What percent of the other 75 questions does she need to answer correctly to score 60% on the entire exam?
  • 217. Solution Required correct answer = 150*60/100 = 90 Questions need to be correct. 80% of 75 questions = 60 q answered correctly. Remaining 30 questions need be correct out of 75 = 30/75*100 = 40
  • 218. Problem - 14 A boy after giving away 80% of his pocket money to one companion and 6% of the remainder to another has 47 paise left with him. How much pocket money did the boy have in the beginning?
  • 219. Solution Let the amount be 100 To the first companion = 100*80/100 = 80 Remaining = 100 – 80 = 20 To the 2nd person = 20*6/100 = 1.20 The remaining = Rs.18.80 or 1880 paise Initial Final 100 1880 X 47 1880x = 47*100 x = 4700/1880 = 2.5
  • 220. Problem - 15 The length of a rectangle is increased by 10% and breath decreased by 10%. Then the area of the new rectangle?
  • 221. Solution I – D – I*D /100 10 -10 – 10*10/100 0 – 1 = -1 Decrease by 1%
  • 223. Profit and Loss • Gain =(S.P.)-(C.P.) • Loss =(C.P.)-(S.P.) • Loss or gain is always reckoned on C.P. • Gain% = [(Gain*100)/C.P.] • Loss% = [(Loss*100)/C.P.] • S.P. = ((100 + Gain%)/100)C.P. • S.P. = ((100 – Loss%)/100)C.P.
  • 224. Problem - 1 A trade man allows two successive discount of 20% and 10%. If he gets Rs.108 for an article. What was its marked price?
  • 225. Solution I1 + I2 – I1*I2/100 20 + 10 – 20*10 /100 = 28% Discount = 28%, 72 Percent Cost is 108 Then 100percent cost = 72 108 100 x 100*108/72 = 150
  • 226. Problem - 2 A trade man bought 500 metres of electric wire at 75 paise per metre. He sold 60% of it at profit of 8%. At what gain percent should he sell the remainder so tas to gain 12% on the whole
  • 227. Solution 500* 60/100 = 300 8 X 12 300 200 300 : 200 = 6 : 4 8 18 12 6 4 Remainder at 18% Profit
  • 228. Problem - 3 A man purchased a box full of pencils at the rate of 7 for Rs. 9 and sold all of them at the rate of 8 for Rs. 11. in this bargains he gains Rs. 10. How many pencils did the box contains.
  • 229. Solution LCM = 7 and 8 = 56 56 pencil cost price = 8*9 = 72 56 Pencil selling price = 7*11 Profit = 77 – 72 = Rs. 5 for 56 pencil Rs. 5 for 56 pencil means , for Rs. 10 the pencils are 112
  • 230. Problem - 4 A cloth merchant decides to sell his material at the cost price, but measures 80cm for a metre. His gain % is?
  • 231. Solution 100 – 80 = 20 cm difference Actual = 80 20/80*100 = 25% Gain
  • 232. Problem - 5 Sales of a book decrease by 2.5% when its price is hiked by 5%. What is the effect on sales?
  • 233. Solution Let the sales be 100 – 2.5 = 97.5 Profit = 100+5 = 105 Sales Profit 97.5 105 100 X 100x = 97.5*105 x = 97.5*105/100 = 102.375 100 – 102.375 = 2.375 = 2.4 profit (app)
  • 234. Problem - 6 A dealer buys a table listed at Rs.1500 and gets successive discount of 20% and 10%. He spends Rs. 20 on transportation and sells it at a profit of 10%. Find the selling price of the table.
  • 235. Solution Discount = 20+10 – 20*10/100 = 28% Actual price = 100 – 28 = 72 100 1500 72 x 72*1500/100 = 1080 Transport = 1080 +20 = 1100 100 1100 110 x 1100*110/100 = 1210
  • 236. Problem - 7 A fridge is listed at Rs. 4000. due to the off season, a shopkeeper announces a discount of 5%. What is the S.P?
  • 238. Problem - 8 If the cost price of 9 pens is equal to the S.P of 11 pens. What is the gain or loss?
  • 239. Solution = 11 – 9 = 2 = 2/11*100 = 18 2/11% loss
  • 240. Problem - 9 A machine is sold for Rs.5060 at a gain of 10% what would have been the gain or loss percent if it had been sold Rs.4370?
  • 241. Solution S.P = Rs.5060 = Gain = 10% C.P = 100/110*5060 = 4600 IF S.P = Rs.4370 and C.P = Rs.4600 Loss = 230 Loss % = 230/4600 * 100 = 5% loss
  • 242. Problem - 10 A person purchased two washing machines each for Rs.9000. he sold one at a loss of 10% and other at a gain of 10%. What is his gain or loss?
  • 243. Solution Each Rs.9000. one is 10% profit and other is 10% loss. So No profit and No loss
  • 244. Problem - 11 Four percent more is gained by selling an article for Rs.180, then by selling if for Rs.175. then its C.P is?
  • 245. Solution Let the cost price = Rs. X 4% of x = 180 – 175 = 4x/100 = 5 4x = 500; x = 500/4 = 125
  • 246. Problem - 12 An article is sold at a profit of 20%. If it had been sold at a profit of 25%. It would have fetched Rs.35% more. The Cost Price of the article is?
  • 247. Solution Let C.P = Rs. X 125% of x – 120% of x = 35 5% of x =Rs.35 = x = 35*100/5 = 700 C.P = Rs. 700
  • 248. Problem - 13 A reduction of 20% in the price of orange enables a man to buy 5 oranges more for Rs. 10. The price of an orange before reduction was,
  • 249. Solution 20% Rs. 10 = Rs.2 Reduced price of 5 oranges = Rs. 2 Reduced price of 1 oranges = 40 p Original price = 40/ 1- 0.20 = 400/8 = 50 Paise
  • 250. Problem - 14 A man sells two horses for Rs.1475. The cost price of the first is equal to the S.P of the second. If the first is sold at 20% loss and the second at 25% gain. What is his total gain or loss? ( in rupees)
  • 251. Solution Let cost price of 1st horse = S.P of 2nd = x C.P of 2nd = S.P of 2nd * 100/125 = x*100/125 = 4x/5 S.P of 1st = C.P of 1st *80/100 = x*80/100 = 4x/5 Neither loss nor gain
  • 252. Problem - 15 Rekha sold a watch at a profit of 15%. Had he bought it at 10% less and sold it for Rs. 28 less, he would have gained 20%. Find the C.P of the Watch.
  • 253. Solution C.P be Rs. X First S.P = 115% of x = 23x/20 and second C.P = 90% x = 9x/10 Second S.P = 120% of 9x/10 = 120/100 * 9x/10 = 27x/25 Given 23x/20 – 27x/25 = 28 = 115x – 108x/100 = 28 7x/100 = 28 = x = 28*100/7 = 400 C.P = Rs.400
  • 255. Probability • Probability: P(є) = n(є) / n(s) • (Addition theorem on probability: n(AUB) = n(A) + n(B) - n(A B) • Mutually Exclusive: P(AUB) = P(A) + P(B) • Independent Events: P(A B) = P(A) * P(B)
  • 256. Problem - 1 Four cards are drawn at random from a pack of 52 playing cards. Find the probability of getting all face cards?
  • 257. Solution n(E) = 52C4 n(S) = 12C4 = 12C4/52C4
  • 258. Problem - 2 Four persons are to be chosen at random from a group of 3 men, 2 women and 4 children. Find the probability of selecting 1 man, 1 woman or 2 children?
  • 259. Solution Total 3 M + 2 W + 4 C = 9 C 4 = 126 n (E) = 3C1 * 2C1 * 4C2 = 36 36/126 = 2/7
  • 260. Problem - 3 A word consists of 9 letters, 5 consonants and 4 vowels. Three letters are chosen at random. What is the probability that more than one vowels will be selected?
  • 261. Solution n(E) = 9C3 = 84 More than one Vowels. So, 2V +1C or 3 V 4C2 *5C1 + 4C3 = 34 = 34/84 = 17/42
  • 262. Problem - 4 A bag contains 10 mangoes out of which 4 are rotten. Two mangoes are taken out together. If one of them was found to be good, then what is the probability that the other one is also good?
  • 263. Solution 10 mangoes – 4 are rotten = 6 good mangoes Getting good mangoes = 6C1/10C1 = 6/10 Getting second mango to be good = 5/9 1st and 2nd mangoes 6/10 *5/9 = 1/3
  • 264. Problem - 5 Out of 13 applicants for a job there are 5 women and 8 men. It is desired to select 2 persons for the job. What is the probability that at least one of the selected person will be a woman?
  • 265. Solution n(E) = 13C2 = 78 n(S) = 1m and 1 w or 2 w = 8C1*5C1 + 5C2 = 50 = 50/78 = 25/39
  • 266. Problem - 6 Two cards are drawn at random from a pack of 52 cards. What is the probability that either both are black or both are queen?
  • 267. Solution P(A) = Both are Black P(B) = Both are Queen P(AnB) = Both are queen and Black P(A) = 26C2/52C2 = 325/1326 P(B) = 4C2 /52C2 = 6/1326 P(AnB) = 2C2 /52C2 = 1/1326 325/1326 + 6/1326 - 1/ 1326 = 55/221
  • 268. Problem -7 A man and his wife appear in an interview for two vacancies in the same post. The probability of husband‟s selection is 1/7 and the probability of wife‟s selection is 1/5. Find the probability that only one of them is selected?
  • 269. Solution Husband‟s Selection = 1/7; Not getting selected = 1 – 1/7 = 6/7 Wife‟s selection = 1/5; Not getting selected = 1 – 1/5 = 4/5 Only one of them is selected = (Husband‟s Selection + Wife Not getting selected) or (Wife‟s selection + Husband‟s Not getting selected) = (1/7*4/5) + 1/5*6/7) = 2/7
  • 270. Problem - 8 Four persons are chosen at random from a group of 3 men, 2 women and 4 children. What is the chance that exactly 2 of them are children?
  • 271. Solution 3 + 2 + 4 = 9C4 = 126 4 members 2(M and W) + 2(boy) 5C2 + 4C2 = 60 = 60 / 126 = 10/21
  • 272. Problem - 9 Prakash can hit a target 3 times in 6 shots, Priya can hit the target 2 times in 6 shots and Akhilesh can hit the target 4 times in 4 shots. What is the probability that at least 2 shots hit the target?
  • 273. Solution Prakash hitting = 3/6; not hitting = 3/6 Priya hitting = 2/6; not hitting = 4/6 Akilesh = 4/4 = 1 At least 2 shots hit target = 3/6*4/6 + 3/6*2/6 = ½
  • 274. Problem - 10 There are two boxes A and B. A contains 3 white balls and 5 black balls and Box B contains 4 white balls and 6 black balls. One box is taken at random and what is the probability that the ball picked up may be a white one?
  • 275. Solution (Box A is selected and a ball is picked up ) or (Box B is selected and a ball is picked up) ½*3/8 + ½*4/10 = 31/80
  • 276. Problem - 11 A bag contains 6 white balls and 4 black balls. Four balls are successively drawn without replacement. What is the probability that they are alternately of different colour?
  • 277. Solution Suppose the balls drawn are in the order white, black, white, black… = 6/10 *4/9*5/8*3/7 = 360/5040 Suppose the balls drawn are in the order black, white, black, white… = 4/10*6/9*3/8*5/7 = 360/5040 360/5040 +360/5040 = 1/7
  • 278. Problem - 12 A problem in statistics is given to four students A, B, C and D. Their chances of solving it are 1/3, ¼, 1/5 and 1/6 respectively. What is the probability that the problem will be solved?
  • 279. Solution A is not solving problem = 2/3, B is not solving problem = ¾ C not solving problem = 4/5 D not solving problem = 5/6 2/3*3/4*4/5*5/6 = 1/3 All together the probability of solving the problem = 1 -1 /3 = 2/3
  • 280. Problem - 13 There are 8 questions in an examination each having only 2 answers choices „Yes‟ or „No‟. All the questions carry equal marks. If a student marks his answer randomly, what is the probability of scoring exacting 50%?
  • 281. Solution Each questions having 2 ways of answering, 1 question = 2!........ 8 question = 2! = 2!*2!*2!*2!*2!*2!*2!*2! = 256 To get 50%, 4 questions need to be correct, 8c4 = 8*7*6*5/1*2*3*4 = 70 = 70/256 = 35/128
  • 282. Problem - 14 A group consists of equal number of men and women. Of them 10% of men and 45% of women are unemployed. If a person is randomly selected from the group find the probability for the selected person to be an employee.
  • 283. Solution Let the number of men is 100 and women be 100 Employed men and women = (100-10)+(100-45) = 145 Probability = 145 / 200 = 29 / 40
  • 284. Problem - 15 The probability of an event A occurring is 0.5 and that of B is 0.3. If A and B are mutually exclusive events. Find the probability that neither A nor B occurs?
  • 285. Solution It is Mutually exclusive events P(A n B)=0 Probability = 1 – ( P(A) + P (B) – P(A n B) ) = 1 – (0.5 + 0.3 – 0) = 0.2
  • 287. Permutation and Combination Permutation means Arrangement Combination means Selection
  • 288. Permutation and Combination • Permutations: Each of the arrangements which can be made by taking some (or) all of a number of items is called permutations. np = n(n-1)(n-2)…(n-r+1)=n!/(n-r)! r • Combinations: Each of the groups or selections which can be made by taking some or all of a number of items is called a combination. nC = n!/(r!)(n-r)! r
  • 289. Types 1. How many ways of Arrangement possible by using word SOFTWARE? SOFTWARE = 8! 2. How many ways of arrangement Possible by using word SOFTWARE, vowels should come together. SFTWR (OAE) = 6! * 3!
  • 290. Types 3. How many ways of Arrangement Possible by using word SOFTWARE, vowels should not come together? SFTWR ( ARE) Not together = Total arrangement – Vowels together = 8! – (5! * 3!)
  • 291. Types 4. How many ways of arrangement possible by using word MACHINE, so that vowels occupy only ODD places. - - - - - - - (7 places) MCHN (AIE) 4 Consonant and 3 vowels. 7 places = 4 ODD places, 3 EVEN places Vowels = 4P3 = 4! Consonant = 4P4 = 4! Total Number of arrangement = 4!*4!
  • 292. Types 5. How many ways of arrangement possible by using word ARRANGEMENT Letter‟s Repetition = 2(A) 2(R) 2 (E) 2 (N) = 11!/2!*2!*2!*2! In a given problem, any letter is repeated more than once that should be divided with total number.
  • 293. Problem - 1 A committee of 5 is to be formed out of 6 gents and 4 ladies. In how many ways this can be done, when at least 2 ladies are included?
  • 294. Solution a. 2 ladies * 3 Gents 4C2 * 6 C3 = 120 b. 3 ladies * 2 Gents 4C3 * 6C2 = 60 c. 4 ladies * 1 Gent 4C4 *6C1 = 1*6 = 6 Total ways = 120 +60 +6 = 186
  • 295. Problem - 2 It is required to seat 5 men and 4 women in a row so that the women occupy the even places. How many such arrangements are possible?
  • 296. Solution Total places = 9 Odd places = 5 Even places = 4 4 even places occupied by 4 women = 4P4 = 4! = 24 5 odd places occupied by 5 men = 5P4 = 5! = 120 Total ways = 120*24 = 2880 ways
  • 297. Problem - 3 A set of 7 parallel lines is intersected by another set of 5 parallel lines. How many parallelograms are formed by this process?
  • 298. Solution Two parallel lines from the first set and any two from the second set will from a parallelogram. 7C2 *5C2 = 21 * 10 = 210
  • 299. Problem - 4 There are n teams participating in a football championship. Every two teams played one match with each other. There were 171 matches on the whole. What is the value of n?
  • 300. Solution Total number of matches played = nC2 nC2 = 171 n(n-1)/2= 171 n2 – n – 342 = 0 (n+18) (n-19) = 0 n = 19
  • 301. Problem - 5 In an examination, a candidate has to pass in each of the 6 subjects. In how many ways can he fail?
  • 302. Solution 6C1 + 6C2 + 6C3 + 6C4+6C5+6C6 1 + 6 + 15 + 20 + 15 + 6 = 63 ways
  • 303. Problem - 6 In how many ways can a pack of 52 cards be distributed to 4 players, 17 cards to each of 3 and one card to the fourth player?
  • 304. Solution 17 cards can be given to 1st player = 52 C17 2nd player = 35C17 3rd player = 18C17 4th player = 1 = 52C17*35C17*18C17 = 52!/17!35! * 35!/17!*18! * 18!/17!*1! = 52!/(17!)3
  • 305. Problem - 7 A foot race will be held on Saturday. How many different arrangements of medal winners are possible if medals will be for first, second and third place, if there are 10 runners in the race …
  • 306. Solution n = 10 r=3 n P r = n!/(n-r)! = 10! / (10-3)! = 10! / 7! = 8*9*10 = 720 Number of ways is 720.
  • 307. Problem - 8 To fill a number of vacancies, an employer must hire 3 programmers from among 6 applicants, and two managers from 4 applicants. What is total number of ways in which she can make her selection ?
  • 308. Solution It is selection so use combination formula Programmers and managers = 6C3 * 4C2 = 20 * 6 = 120 Total number of ways = 120 ways.
  • 309. Problem - 9 A man has 7 friends. In how many ways can he invite one or more of them to a party?
  • 310. Solution In this problem, the person is going to select his friends for party, he can select one or more person, so addition = 7C1 + 7C2+7C3 +7C4 +7C5 +7C6 +7C7 = 127 Number of ways is 127
  • 311. Problem - 9 Find the number of different 8 letter words formed from the letters of the word EQUATION if each word is to start with a vowel
  • 312. Solution For the words beginning with a vowel, the first letter can be any one of the 5 vowels, the remaining 7 places can be filled by 7P7 = 5040 The number of words = 5 * 5040 = 25200
  • 313. Problem - 10 In how many different ways can the letters of the word TRAINER be arranged so that the vowels always come together?
  • 314. Solution A,I,E can be arranged in 3! Ways (5! * 3!) / 2! = 360 ways
  • 315. Problem - 11 In how many different ways can the letters of the Word DETAIL be arranged so that the vowels may occupy only the odd positions?
  • 316. Solution ___ ___ ___ ___ ___ ___ 3P3 = 3! = 1*2*3 = 6 3P3 = 3! = 1*2*3 = 6 = 6*6 = 36
  • 317. Problem - 12 There are 5 red, 4 white and 3 blue marbles in a bag. They are taken out one by one and arranged in a row. Assuming that all the 12 marbles are drawn, find the number of different arrangements?
  • 318. Solution Total number of balls = 12 Of these 5 balls are of 1st type (red), 4 balls are the 2nd type and 3 balls are the 3rd type. Required number of arrangements = 12!/5!*4!*3! = 27720
  • 319. Problem - 13 5 men and 5 women sit around a circular table, the en and women alternatively. In how many different ways can the seating arrangements be made?
  • 320. Solution 5 men can be arranged in a circular table in 4 ways = 24 ways There are 5 seats available for 5 women they can be arranged in 5 ways No. of ways = 5!*4! = 2880 ways
  • 321. Problem - 14 In a chess board there are 9 vertical and 9 horizontal lines. Find the number of rectangles formed in the chess board.
  • 322. Solution Solution: 9C2 * 9C2 = 1296
  • 323. Problem - 15 In how many ways can a cricket team of 11 players be selected out of 16 players, If one particular player is to be excluded?
  • 324. Solution Solution: If one particular player is to be excluded, then selection is to be made of 11 players out of 15. 15C11= 15!/( 11!*4!)=1365 ways
  • 326. Area and Volume Cube: • Let each edge of the cube be of length a. then, • Volume = a3cubic units • Surface area= 6a2 sq.units. • Diagonal = √3 a units.
  • 327. Area and Volume Cylinder: • Let each of base = r and height ( or length) = h. • Volume = πr2h • Surface area = 2 πr h sq. units • Total Surface Area = 2 πr ( h+ r) units.
  • 328. Area and Volume Cone: • Let radius of base = r and height=h, then • Slant height, l = √h2 +r2 units • Volume = 1/3 πr2h cubic units • Curved surface area = πr l sq.units • Total surface area = πr (l +r)
  • 329. Area and Volume Sphere: • Let the radius of the sphere be r. then, • Volume = 4/3 πr3 • Surface area = 4 π r2sq.units
  • 330. Area and Volume Circle: A= π r 2 Circumference = 2 π r Square: A= a 2 Perimeter = 4a Rectangle: A= l x b Perimeter= 2( l + b)
  • 331. Area and Volume Triangle: A = 1/2*base*height Equilateral = √3/4*(side)2 Area of the Scalene Triangle S = (a+b+c)/ 2 A = √ s*(s-a) * (s-b)* (s-c)
  • 332. Problem - 1 A rectangular sheet of size 88 cm * 35 cm is bent to form a cylindrical shape with height 35 cm. What is the area of the base of the cylindrical shape?
  • 333. Solution The circumference of the circular region = 88 cm 2 r = 88 r = 88*7/22*2 = 14 cm Area of the base = r2 = 22/7*14*14 v= 616 cm2
  • 334. Problem - 2 The radius of the base of a conical tent is 7 metres. If the slant height of the tent is 15 metres, what is the area of the canvas required to make the tent?
  • 335. Solution R=7m L = 15 m Area of Canvas required = Curved Surface Area of cone rl = 22/7*7*15 = 330 sq.m
  • 336. Problem - 3 Three spherical balls of radius 1 cm, 2 cm and 3 cm are melted to form a single spherical ball. In the process, the material loss was 25%. What would be the radius of the new ball?
  • 337. Solution Vol. of sphere = 4/3 r3 Vol. of 3 small spherical balls = 4/3 ( 13+23+33) = 4/3 (1+8+27) = 4 /3 (36) = 48 Material loss = 25% Vol. of the single spherical ball = 48 *75/100 = 48 * ¾ = 36 V = 4/3 r3 = 36 r3 = 36*3/4 = 27 r = 3 cm
  • 338. Problem - 4 A rectangular room of size 5m(l)*4m(w)*3m(h) is to be painted. If the unit of painting is Rs. 10 per sq.m, what is the total cost of painting?
  • 339. Solution Area of 4 walls = 2h(l+b) The area to be painted includes the 4 walls and the top ceiling. Area to be painted = 2h (l+b) +lb = 2*3 (5+4) + 5*4 = 54+20 = 74 sq.m. Total cost of painting = 74*10 = Rs.740
  • 340. Problem - 5 The radius of a sphere is r units. Each of the radius of the base and the height of a right circular cylinder is also r units. What is the ratio of the volume of the sphere to that of the cylinder?
  • 341. Solution Vol. of sphere = 4/3 r3 and Vol. of Cylinder = r2h = r3 Required Ratio = 4/3 r3: r3 = 4/3 : 1 =4:3
  • 342. Problem - 6 A measuring jar of internal diameter 10 cm is partially filled with water. Four equal spherical balls of diameter 2 cm each, are dropped in it and they sink down in the water completely. What will be the increase in the level of water in the jar?
  • 343. Solution Radius of each ball = 1 cm Vol. of 4 balls = 4* 4/3 (r)3 = 16/3 cm3 Vol. of water raised in the Jar = Vol. of 4 balls Let h be the rise in water level, then Area of the base *h = 16/3 *5*5*h = 16/3 H = 16/3*25 = 16/75 cm
  • 344. Problem - 7 What is the cost of planting the field in the form of the triangle whose base is 2.8 m and height 3.2 m at the rate of Rs.100 / m2
  • 345. Solution Area of triangular field = ½ * 3.2 * 2.8 m2 = 4.48 m2 Cost = Rs.100 * 4.48 = Rs.448..
  • 346. Problem - 8 Find the length of the longest pole that can be placed in a room 14 m long, 12 m broad, and 8 m high.
  • 347. Solution Length of the longest pole = Length of the diagonal of the room = √(142 + 122 + 82) = √ 404 = 20.09 m
  • 348. Problem - 9 Area of a rhombus is 850 cm2. If one of its diagonal is 34 cm. Find the length of the other diagonal.
  • 349. Solution 850 = ½ * d1 * d2 = ½ * 34 * d2 = 17 d2 d2 = 850 / 17 = 50 cm Second diagonal = 50cm
  • 350. Problem - 10 A grocer is storing small cereal boxes in large cartons that measure 25 inches by 42 inches by 60 inches. If the measurement of each small cereal box is 7 inches by 6 inches by 5 inches then what is maximum number of small cereal boxes that can be placed in each large carton ?
  • 351. Solution No. of Boxes = 25*42*60 / 7*6*5 = 300 300 boxes of cereal box can be placed.
  • 352. Problem - 11 If the radius of a circle is diminished by 10%, what is the change in area in percentage?
  • 353. Solution = x + y + xy/100 = -10 - 10 + 10*10/100 = -19% Diminished area = 19%.
  • 354. Problem - 12 A circular wire of radius 42 cm is bent in the form of a rectangle whose sides are in the ratio of 6:5. Find the smaller side of the rectangle?
  • 355. Solution length of wire = 2 πr = (22/7*14*14)cm = 264cm Perimeter of Rectangle = 2(6x+5x) cm = 22xcm 22x =264 x = 12 cm Smaller side = (5*12) cm = 60 cm
  • 356. Problem - 13 A beam 9m long, 40cm wide and 20cm deep is made up of iron which weights 50 kg per cubic metre. Find the weight of the Beam.
  • 357. Solution Vol. of the Beam = lbh = 9*40/100*10/100 = 72 m3 Weight of the iron beam is given as lm3 = 50 kg 72/100 m3 = 72/100*50 = 36 kg
  • 358. Problem - 14 If the length of a rectangle is reduced by 20% and breadth is increased by 20%. What is the percentage change in the area?
  • 359. Solution x + y + (xy/100)% = - 20 + 20 – 400/100 = -4 The area would decrease by 4%
  • 360. Problem - 15 Find the number of bricks measuring 25 cm in length, 5 cm is breadth and 10 cm in height for a wall 40 m long, 75 cm broad and 5 metres in height?
  • 361. Solution Vol. of the wall = 40*72/100*5 = 150 m3 Vol. of 1 bricks = 25/100*5/100*10/100 = 1/80 m3 Number of bricks required = 150/1/800 = 150*800 = 120000
  • 363. Calendar Odd days: 0 = Sunday 1 = Monday 2 = Tuesday 3 = Wednesday 4 = Thursday 5 = Friday 6 = Saturday
  • 364. Calendar Month code: Ordinary year J=0 F=3 M=3 A=6 M=1 J=4 J=6 A=2 S=5 O=0 N=3 D=5 Month code for leap year after Feb. add 1.
  • 365. Calendar Ordinary year = (A + B + C + D )-2 -----------------------take remainder 7 Leap year = (A + B + C + D) – 3 ------------------------- take remainder 7
  • 366. Problem - 1 11th January 1997 was a Sunday. What day of the week on 7th January 2000?
  • 367. Solution 11th Jan 1997 = Sunday 11th Jan 1998 = Monday 11th Jan 1999 = Tuesday 11th Jan 2000 = Wednesday 7th Jan 2000 is on Saturday
  • 368. Problem - 2 What day of the week was on 5th June 1999?
  • 369. Solution A+B+C+D – 2 / 7 A = 1999/7 = 4 B = 1999/4 = 499/7 = 2 C = June = 4 D = 5/7 = 5 = 4+2+4+5 – 2/7 = 13/7 = 5 = Saturday
  • 370. Problem - 3 On what dates of August 1988 did Friday fall?
  • 371. Solution A = 1988 / 7 = 0 B = 1988/4 = 497/7 = 0 C=3 D=x 0+0+3+x+3/7 = x/7 = 5(Friday) Friday falls on = 5,12,19,26
  • 372. Problem - 4 India got independence on 15 August 1947. What was the day of the week?
  • 373. Solution A = 1947/7 = 1 B = 1947/4 = 486/7 = 3 C = 15/7 = 1 D=2 1+3+1+2 – 2 /7 = 5/7 = Friday
  • 374. Problem - 5 7th January 1992 was Tuesday. Find the day of the week on the same date after 5 years. i.e on 7th January 1997.
  • 375. Solution 7th January 1992 = Tuesday 7th January 1993 = Thursday (Leap) 7th January 1994 = Friday 7th January 1995 = Saturday 7th January 1996 = Monday ( Leap) 7th January 1997 = Tuesday
  • 376. Problem - 6 The first Republic day of India was celebrated on 26th January 1950. What was the day of the week on that date?
  • 377. Solution A = 1950/7 = 4 B = 1950/4 = 487/7 = 4 C=0 D = 26/7 = 5 4+4+0+5 – 2/7 = 11/7 = 4 = Thursday
  • 378. Problem - 7 Find the Number of times 29th day of the month occurs in 400 consecutive year?
  • 379. Solution 1 year = 1 (Ordinary Year) 1 year = 12 (Leap Year) 400 years = 97 leap year 97 * 12 = 1164 303*11 = 3333 = 1164+3333 = 4497 times
  • 380. Problem - 8 If 2nd March 1994 was on Wednesday, 25 Jan 1994 was on,
  • 381. Solution A = 1994/7 = 6 B = 1994/4 = 498/7 = 1 C=0 D = 25/7 = 4 = 6 + 1 + 0 + 4 – 2 / 7 = 3 = Tuesday
  • 382. Problem - 9 Calendar for 2000 will serve also?
  • 383. Solution = 2000 + 2001 + 2002 + 2003 + 2004 = 2 + 1 + 1 + 1 + 2 = 7 (Complete Week) 2005
  • 384. Problem - 10 If Pinky‟s 1st birthday fell in Jan 1988 on one of the Monday‟s, the day on which are was born is,
  • 385. Solution Jan = 1988 = Monday Jan = 1987 = Sunday
  • 386. Problem - 11 Akshaya celebrated her 60th birthday on Feb 24, 2000. What was the day?
  • 387. Solution A = 2000 /7 = 7 B = 2000/4 = 500/7 = 3 C=3 D = 24/7 = 0 = 7+3+3+0-3/7 = 10/7 = 3 = Wednesday
  • 388. Problem - 12 On what dates of April 2008 did Sunday Fall?
  • 389. Solution Calculate for 1st April 2008 A = 2008/7 = 6 B = 2008/4 = 502/7 = 5 C = 1/7 = 1 D=0 = 6+5+1+0 – 3/ 7 = 2 = Tuesday 1st April on Tuesday, then 1st Sunday fall on 6. Sunday falls on 6, 13, 20, 27.
  • 390. Problem - 13 Today is Friday. After 62 days it will be,
  • 391. Solution 62 / 7 = 6 days after Friday then it will be Tuesday
  • 392. Problem - 14 What will be the day of the week on 1st Jan 2010?
  • 393. Solution A=1 B=5 C=0 D=1 = 1+5+0+1 – 2/ 7 = 5/7 = 5 = Friday
  • 394. Problem - 15 What is the day of the week on 30/09/2007?
  • 395. Calendar Solution: A = 2007 / 7 = 5 B = 2007 / 4 = 501 / 7 = 4 C = 30 / 7 = 2 D=5 ( A + B + C + D )-2 = ----------------------- 7 = ( 5 + 4 + 2 + 5) -2 ----------------------- = 14/7 = 0 = Sunday 7
  • 396. Clock
  • 397. Clocks Clock: Angle between hour hand and minute hand = (11m/2) – 30h Angle between minute hand and hour hand =30h – (11m/2)
  • 398. Problem - 1 What is the angle between the minute hand and hour hand when the time is 2.15?
  • 399. Solution = 11 m/2 – 30(h) = 11 15/2 – 30(2) = 11(7.5) – 60 = 82.5 – 60 = 22 1/2
  • 400. Problem - 2 At what time between 5 and 6 o‟clock the hands of a clock coincide?
  • 401. Solution Coinciding Angle = 0 Min. hand to hour hand = 25 min apart 60/55*25 = 12/11 * 25 = 300/11 = 27 3/11min past 5
  • 402. Problem - 3 At what time between 12 and 1 o‟clock both the hands will be at right angles?
  • 403. Solution Right angle = 90 degrees = 30(h) – 11 m/2 90 = 30(12) – 11 m/2 180 = 360 – 11m 11m = 360 – 180 M = 180/11 16 4/11 past 12
  • 404. Problem - 4 Find at what time between 7 and 8 o‟clock will the hands of a clock be in the same straight line but not together?
  • 405. Solution Minute hand to hour hand = 35 min apart Straight line not together = 30 min apart Difference = 35 – 30 = 5 min = 60/55*5 = 12/11*5 = 60/11 = 55 5 / 11 past 7
  • 406. Problem - 5 At what time between 5 and 6 are the hands of the clock 7 minutes apart?
  • 407. Solution 7 min space behind the hour hand: 25 min – 7 min = 18 min 60/55 *18 = 216/11 = 19 7/11 min past 5 7 Min space ahead the hour hand 25 min + 7 min = 32 min 60/55*32 = 12/11*32 = 384/11 = 34 10/11 min past 5
  • 408. Problem - 6 A clock strikes 4 and takes 9 seconds. In order to strike 12 at the same rate what will be the time taken?
  • 409. Solution Strike Sec 3 (interval) 9 11 x 3x = 11*9 X = 11*9/3 = 33 Sec